Download as pdf or txt
Download as pdf or txt
You are on page 1of 64

DO NOT OPEN THIS TEST BOOKLET UNTIL YOU ARE ASKED TO DO SO

PTS2024/AIOMT/102023/07

TEST BOOKLET

ALL INDIA OPEN MOCK TEST


GENERAL STUDIES PAPER - I
Time Allowed: Two Hours Maximum Marks: 200

INSTRUCTIONS

1. IMMEDIATELY AFTER THE COMMENCEMENT OF THE EXAMINATION, YOU SHOULD CHECK THAT
THIS TEST BOOKLET DOES NOT HAVE ANY UNPRINTED OR TORN OR MISSING PAGES OR ITEMS,
ETC. IF SO, GET IT REPLACED BY A COMPLETE TEST BOOKLET.
2. Please note that it is the candidate’s responsibility to encode and fill in the Roll Number carefully
without any omission or discrepancy at the appropriate places in the OMR Answer Sheet. Any
omission/discrepancy will render the Answer Sheet liable for rejection.
3. You have to enter your Roll Number on the test booklet in the Box provided
alongside. DO NOT write anything else on the Test Booklet.
4. This Test Booklet contains 100 items (questions). Each item is printed in English. Each item comprises four
responses (answers). You will select the response which you want to mark on the Answer Sheet. In case you
feel that there is more than one correct response, mark the response which you consider the best. In any case,
choose ONLY ONE response for each item.
5. You have to mark all your responses ONLY on the separate Answer Sheet provided. See directions in the
Answer Sheet.
6. All items carry equal marks.
7. Before you proceed to mark in the Answer Sheet in response to various items in the Test Booklet, you have to
fill in some particulars in the Answer Sheet as per instructions sent to you with your Admission Certificate.
8. After you have completed filling in all your responses on the Answer Sheet and the examination has concluded,
you should hand over to the Invigilator only the Answer Sheet. You are permitted to take away with you the
Test Booklet.
9. Sheets for rough work are appended in the Test Booklet at the end.
10. Penalty for wrong answers:
THERE WILL BE PENALTY FOR WRONG ANSWERS MARKED BY A CANDIDATE IN THE OBJECTIVE
TYPE QUESTION PAPERS.
(i) There are four alternatives for the answer to every question. For each question for which a wrong answer
has been given by the candidate, one-third of the marks assigned to that question will be deducted as
penalty.
(ii) If a candidate gives more than one answer, it will be treated as a wrong answer even if one of the given
answers happens to be correct and there will be same penalty as above to that question.
(iii) If a question is left blank, i.e., no answer is given by the candidate, there will be no penalty for that
question.

DO NOT OPEN THIS TEST BOOKLET UNTIL YOU ARE ASKED TO DO SO

PTS2024/AIOMT/102023/07
1. Regarding the Respiratory syncytial virus 4. Consider the following statements regarding
(RSV), consider the following statements: Angiotensin Converting Enzyme 2 (ACE2):
1. RSV is a single-stranded RNA virus.
Statement-I:
2. Its name is derived from the large cells
known as syncytia that form when ACE2 is a functional receptor for the spike
infected cells fuse. glycoprotein of the human coronaviruses
3. It cannot survive on hard surfaces for a such as SARS-CoV-2, the causative agent of
longer duration. coronavirus disease-2019.
4. It is more serious for infants and older Statement-II:
adults.
ACE2 is known to be expressed exclusively
How many of the above statements is/are on the membrane of cells of various organs
correct?
such as intestine, kidney, etc.
(a) Only one
Which one of the following is correct in
(b) Only two respect of the above statements?
(c) Only three
(a) Both Statement-I and Statement-II are

RE
(d) All four correct and Statement-II is the correct
explanation for Statement-I.
2. Consider the following statements regarding
the Open Radio Access Network (O-RAN): (b) Both Statement-I and Statement-II
are correct and Statement-II is not the
1. An O-RAN is a totally disaggregated
correct explanation for Statement-I.
approach to deploying mobile networks
built entirely on cloud native principles. (c) Statement-I is correct but Statement-II
2. The future goal for O-RAN in the
O
cellular network is to interoperate both
seamlessly and securely regardless of its
originating vendor cloud-native
is incorrect.
(d) Statement-I is incorrect but Statement-
II is correct.
SC
Which of the above statements is/are
incorrect? 5. Which of the following statement make the
blue straggler stars different from the other
(a) 1 only
stars?
(b) 2 only
(a) Many blue stragglers are formed through
(c) Both 1 and 2
mass transfer from a close binary
(d) Neither 1 nor 2 companion star.
GS

(b) They run out of fuel (Hydrogen) much


3. Consider the following statements:
earlier than their siblings in their due
1. The Indian Biological Data Centre is the course of life.
first national repository for life science
data in the country. (c) They are smaller than the rest of the
2. The Indian Crop Phenome Database is stars due to the peculiarity in their
a domain of the Indian Biological Data formation.
Centre, developed for the digitization of
crop phenome data. (d) They are not a part of globular clusters
due to their large mass.
3. Both IBDC and ICPD are supported by
the Government of India through the
Department of Biotechnology. 6. Consider the following pairs:
How many of the statements given above are 1. Leptons - Absence of colour charge
correct?
2. Quarks - A type of fermions
(a) Only one
3. Higgs Boson - Explaining particles
(b) Only two attainment of their masses
(c) All three 4. Gluons - Bosons that carry the weak
(d) None nuclear force

3 PTS2024/AIOMT/102023/07
How many of the above pairs is/are correctly 3. Australian – Joint sitting of
matched? Constitution the two Houses of
Parliament
(a) Only one
4. Russia – Ideals of liberty,
(b) Only two
Constitution equality, and
(c) Only three fraternity
(d) All four How many of the pairs given above is/are
matched correctly?
7. Consider the following pairs: (a) All four
Polymers Corresponding (b) Only three
monomers
(c) Only two
1. Bakelite – Phenol and
(d) Only one
Formaldehyde
2. BUNA – N – Vinyl Cyanide and 1,
10. The Code on Wages, 2020 mentions the
3-butadiene

RE
concept of a floor wage, which empowers the
3. Teflon – Tetrafluoro Ethane central government to fix ‘floor wages’ taking
4. Rubber – Isoprene into account the minimum ‘living standards
of workers.’ In this context, which of the
How many of the above pairs is/are correctly following principle secure a living wage for
matched? all workers?
(a) Only one (a) Gandhian Principles
(b) Only two
O (b) Liberal-Intellectual Principles
(c) Only three (c) Socialistic Principles
(d) All four
SC
(d) None of them

8. Consider the following: 11. With respect to the procedure for the
1. Diatoms amendment of the Indian constitution, which
of the following statements is/are correct?
2. Dinoflagellates
1. The formation of new states, alteration
3. Amoebas
of areas and boundaries can only be
GS

4. Saccharomyces cerevisiae (Baker’s amended by a special majority of the two


yeast) Houses of Parliament.
5. Blue-green algae 2. Fundamental Rights and Directive
Which of the above belong to the “Protists”? Principles of State Policy can be
amended by the simple majority of the
(a) 1, 2 and 3 only Parliament.
(b) 2, 4, and 5 only Select the correct answer using the code
(c) 1, 3 and 5 only given below:

(d) 1, 2, 3 and 4 only (a) 1 only


(b) 2 only
9. Consider the following pairs: (c) Both 1 and 2
Sources Feature (d) Neither 1 nor 2
Borrowed
1. Japanese – Procedure 12. Consider the following statements regarding
Constitution established by Law the office of Governor:
2. French – Fundamental Duties 1. The Governor has security of tenure and
Constitution a fixed term of office.

PTS2024/AIOMT/102023/07 4
2. A decision to remove a Governor can be 15. Article 263 of the Constitution of India
challenged in a court of law. provides for the establishment of an Inter-
State Council. In this context, which of the
3. A governor whose term has expired following is incorrect regarding the duty of
cannot be reappointed in the same state this council?
or any other state.
(a) Inquiring into and advising upon
How many of the above statements is/are disputes which may have arisen between
incorrect? States.
(a) Only one (b) Investigating and discussing subjects
in which some or all the States, or the
(b) Only two Union and one or more of the States,
(c) All three have a common interest.

(d) None (c) Making recommendations upon any


subject and recommendations for the
better coordination of policy and action.
13. Regarding the powers and privileges of (d) The public interests would not be served
Members of Parliament (MPs), consider the by the establishment of a Council.
following statements:

RE
1. Parliamentary privileges are special 16. It is found in regions of the country which
rights, immunities and exemptions receive heavy rainfall with an alternate
enjoyed by the two Houses of dry and wet period. They are formed from
Parliament. the decomposition of rocks and contain iron
oxide, which gives them red or pink color.
2. The members enjoy freedom from arrest This soil is ordinarily deficient in nitrogen
in any civil case 40 days before and after and is weak in lime content. It is an acidic
the adjournment of the house and when

3.
the house is in session.
The members of the house have the
power and right to exclude strangers
O
who are not members of the house from
soil.
Which of soils given below is described in the
above passage?
(a) Peat soil
SC
the proceedings. (b) Alluvial soil
How many of the statements given above are (c) Laterite soil
correct? (d) Black soil
(a) Only one
17. Consider the following pairs:
(b) Only two
(c) All three Plates Location
GS

(d) None 1. Cocos plate - Between Indian


and Australian
plates
14. Consider the following statements:
2. Nazca plate - Between South
1. The Judges of the Jammu & Kashmir America and
High Court are to be appointed by the Pacific plate
President.
3. Capricorn - Between Central
2. Appointment of permanent judges in a microplate America and
high court having jurisdiction over more Pacific plate
than one state, the President would
4. Caroline plate - Between the
initiate proposal to the Governor of the
Philippine and
two states concerned.
Indian plate
Which of the above given statements is/are
How many of the pairs given above is/are
correct? correctly matched?
(a) 1 only (a) Only one
(b) 2 only (b) Only two
(c) Both 1 and 2 (c) Only three
(d) Neither 1 nor 2 (d) All four

5 PTS2024/AIOMT/102023/07
18. Consider the following statements regarding Select the correct option from the code given
western disturbances: below:
1. The western cyclonic disturbances (a) 1, 2 and 4 only
experienced in the north and north-
(b) 1, 2, 3 and 5 only
western parts of the country are brought
in by the subtropical easterly jet (c) 3, 4 and 5 only
stream.
(d) 1, 2, 3, 4 and 5
2. The sub-tropical westerly jet stream
blows over Peninsular India. 21. Consider the following statements:
3. Western disturbances are extra-tropical 1. Odisha is India’s largest bauxite
cyclones developed in the Arabian Sea. producer followed by Gujarat and
Which of the statements given above is/are Jharkhand.
incorrect? 2. The two main types of ore found in India
(a) 1 and 2 only are haematite and limonite.

(b) 3 only 3. Balangir is an active mining centre with

RE
several graphite grades.
(c) 2 and 3 only
4. Salal-Haimana area of the Reasi district
(d) 1, 2 and 3 in Jammu and Kashmir is rich in lithium
deposits.
19. Consider the following: How many of the above statements is/are
incorrect regarding mineral and metal

1.
River

Machchhu river -
State

Gujarat
O deposits in India?
(a) Only one
(b) Only two
2. Lukha river - Meghalaya
SC
(c) Only three
3. Bhogdoi river - Assam (d) All four

4. Parbati river - Madhya


22. Consider the following statements:
Pradesh
Statement-I:
How many of the pairs given above is/are
The Peninsular plateau is a tableland and is
correctly matched?
GS

the part of oldest landmass.


(a) 1 and 3 only
Statement-II:
(b) 2 and 4 only
Peninsular plateau was formed due to the
(c) 1, 2 and 3 only breaking and drifting of the Gondwana
(d) 1, 2, 3 and 4 land.
Select the correct answer using the code
20. Which of the following factors influence given below:
Indian Monsoon? (a) Both Statement-I and Statement-II are
1. Differential heating and cooling of land correct and Statement-II is the correct
and water. explanation for Statement-I.
2. Shift of the position of Inter Tropical (b) Both Statement-I and Statement-II
Convergence Zone (ITCZ) are correct and Statement-II is not the
3. Presence of the high-pressure area in correct explanation for Statement-I.
east of Madagascar (c) Statement-I is correct but Statement-II
4. Cooling of Tibetan plateau during is incorrect.
summer seasons (d) Statement-I is incorrect but Statement-
5. Westerly jet stream II is correct.

PTS2024/AIOMT/102023/07 6
23. Which of the following taxes are applied 2. For inferior goods, demand decreases
on petrol/diesel by Central and State with an increase in consumer’s income.
governments? 3. Veblen goods are those for which demand
1. Excise duties increases with an increase in price.
2. Value-added tax or a sales tax How many of the statements given above is/
3. Goods and services tax are correct?
Select the correct answer using the code (a) Only one
given below: (b) Only two
(a) 1 and 3 only (c) All three
(b) 2 and 3 only (d) None
(c) 1 and 2 only
(d) 1, 2 and 3 27. Consider the following statements:
1. Foreign investment in India is primarily
24. In the context of the Indian economy, consider regulated through the Foreign Exchange

RE
the following: Management Act, 1999.
1. Acquisition of produced assets 2. As per present norms, any investment
coming from an investment vehicle
2. Purchases of second-hand assets by
whose sponsor is owned by person
producers
resident outside India will be counted
3. Investments by resident producers under FDI.
4. Disposal of fixed assets 3. Asset Reconstruction Companies are
How many of the above are included in the
Gross Fixed Capital Formation (GFCF)?
(a) Only one
O allowed to receive FDI but only after
government’s approval.
Which of the statements given above is/ are
correct?
SC
(b) Only two
(a) Only one
(c) Only three
(b) Only two
(d) All four
(c) All three
25. With reference to Participatory Notes, (d) None
consider the following statements:
GS

1. These are financial instruments required 28. With reference to existing regulation on
to invest in Indian securities without crypto assets in India, which one of the
registering with the Securities and following statement is incorrect?
Exchange Board of India (SEBI).
(a) All crypto products are regulated by
2. The investments through P-notes are Reserve Bank of India (RBI).
considered as Offshore Derivative
(b) All crypto exchanges are registered
Investments (ODIs).
as company under the Companies act,
Which of the statements given above is/are 2013.
correct? (c) GST is applicable on both crypto assets
(a) 1 only as well as crypto exchanges.
(b) 2 only (d) Mining of crypto assets is prohibited
in India and only the transfer and
(c) Both 1 and 2 transaction of assets is allowed.
(d) Neither 1 nor 2
29. With reference to the Wholesale Price Index
26. Consider the following statements: (WPI) and Consumer Price Index (CPI),
consider the following statements:
1. Giffen goods are those goods for which
demand decreases with an increase in 1. Unlike the WPI basket, CPI basket
price. includes services.

7 PTS2024/AIOMT/102023/07
2. Fuel and Power component has the 32. Consider the following statements:
maximum weightage in the WPI while 1. The Himalayan Moist Temperate
the Food and Beverages component holds Forests have mixed species of broad-
the maximum weightage in the CPI. leafed evergreen trees and conifers.
3. WPI helps in timely intervention by
2. The boreal forests are the earth’s
the Government to check inflation in
southernmost and are characterized by
essential items while CPI is used as
a high diversity of tree species.
an anchor for the inflation targeting in
India. 3. Delta of Mahanadi are covered by
4. Unlike the WPI basket, CPI includes Mangrove Forests.
indirect taxes. How many of the above statements is/
How many of the statements given above is/ are incorrect regarding forest types and
are correct? locations?
(a) Only one (a) Only one
(b) Only two (b) Only two
(c) Only three (c) All three

RE
(d) All four
(d) None

30. With reference to world economy, consider


the following statements: 33. Consider the following:

1. An anti-dumping duty is a protectionist 1. Ice-albedo feedback


tariff that the domestic government 2. Lapse rate feedback
imposes to neutralize the impact of 3. Water vapour feedback

2.
import subsidies.
O
Countervailing duty is a specific form
4. Ocean heat transport
of duty that the domestic government How many of the above given are the causes
imposes on foreign imports. of arctic amplification?
SC
Which of the statements given above is/are (a) Only one
correct? (b) Only two
(a) 1 only (c) Only three
(b) 2 only (d) All four
(c) Both 1 and 2
(d) Neither 1 nor 2 34. Consider the following statements regarding
GS

solar storms:
31. How many of the following measures can be 1. The solar wind conditions that are
taken to check Demand-Pull Inflation? effective for creating geomagnetic storms
are sustained periods of high-speed solar
1. Reducing the money, the government is wind and a southward directed solar
pumping into the economy, especially wind magnetic field.
consumptive type.
2. Stealth Coronal mass ejections (CMEs)
2. Central Bank following a Dear Money exhibit signatures at the solar surface
Policy. but showed up in the lower corona.
3. Increasing interest rate on loans. 3. In space, a ring of westward current
4. Increasing imports of goods which are around Earth produces electrical
short in supply. disturbances on the ground.
Select the correct option using the code given Which of the given statements is/are
below: incorrect?
(a) Only one (a) 1 only
(b) Only two (b) 2 and 3 only
(c) Only three (c) 1 and 2 only
(d) All four (d) 2 only

PTS2024/AIOMT/102023/07 8
35. Consider the following statements regarding
Ramsar sites in India: 2. Halosere - Succession of
microorganism on
1. The Tampara lake, a Ramsar site, is a dead matter.
freshwater lake in Madhya Pradesh.
2. Tampara lake is an important habitat 3. Eosere - Development of
for endangered species such as cyprinus vegetation in an era.
carpio and river tern. 4. Senile - Succession starting
Which of the above given statements is/are in saline soil or
incorrect? water.
(a) 1 only How many of the above pairs is/are correctly
(b) 2 only matched about ecological succession?
(c) Both 1 and 2 (a) Only one
(d) Neither 1 nor 2 (b) Only two
(c) Only three
36. Match the following pairs:
(d) All four

RE
Ecological Description
terms 38. Which of following statements regarding
endemism is incorrect?
1. Biomagnification A. Maximum
number of (a) Paleo-endemism in a species can arise
individuals of a when a species going extinct in other
species that the regions.
environment

2. Carrying
capacity
B.
sustain.
Build-up of
nitrogen and
phosphorus
O
can carry and (b) Being endemic to a small area reflects a
threat to the survival of a species.
(c) Lion-tailed macaque is endemic to the
Western Ghats of India.
SC
in aquatic (d) Endemism shows the species richness in
environment a particular area.
3. Niche C. Accumulation of
pesticides and 39. Consider the following statements:
pollutants in
organisms Statement-I:
4. Eutrophication D. Range of Thermal shocks due to the closing down of
GS

resources and power plant can kill fishes in water bodies.


conditions
allowing Statement-II:
the species
to maintain Sudden increase or decrease of temperature
a viable of water bodies leads to thermal pollution.
population.
Select the correct option from the code given
Select the correct answer using the code below:
given below:
(a) Both Statement-I and Statement-II are
(a) 1-B, 2-D, 3-C, 4-A
correct and Statement-II is the correct
(b) 1-A, -B, 3-D, 4-C
explanation for Statement-I.
(c) 1-D, 2-B, 3-A, 4-C
(d) 1-C, 2-A, 3-D, 4-B (b) Both Statement-I and Statement-II
are correct and Statement-II is not the
37. Consider the following: correct explanation for Statement-I.
(c) Statement-I is correct but Statement-II
Types of Description is incorrect.
Seres
(d) Statement-I is incorrect but Statement-
1. Psammosere - Succession initiating II is correct.
on sandy areas.

9 PTS2024/AIOMT/102023/07
40. With reference to Kanaganahalli Site, Which of the above statements is/are
consider the following statements: correct?
1. Kanaganahalli was a small and (a) 1 only
ordinary village on the bank of Bhima in (b) 2 onl
Karnataka.
(c) Both 1 and 2
2. Its excavation includes the stone portrait
of Mauryan Emperor Ashoka with the (d) Neither 1 nor 2
inscription “Raya Asoko” in Kharosthi
script on it 43. Consider the following statements:
3. A Maha Stupa has been discovered at 1. Indian Copyright Act, 1957 seeks to
this site which was referred to as the provide for the registration and better
Great Stupa of the netherworlds. protection of geographical indications
relating to goods in India.
How many of the above statements is/are
2. The GI tag is valid for a period of 20 years
correct?
following which it cannot be renewed.
(a) Only one 3. White onion (Maharashtra) and Mithila

RE
(b) Only two Makhana (Bihar) have recently been
given the GI Tag.
(c) All three
How many of the above statements is/are
(d) None correct?
(a) Only one
41. Consider the following statements regarding
(b) Only two
India’s postal code:
1.
O
The six-digit pin code used by India Post (c) All three
to deliver mails was created on by Jaglal (d) None
Choudhary.
SC
2. The first digit of the six-digit postal code 44. Consider the following statements:
indicates the sub-zone, that is, states 1. In Warli painting, the goddess Palaghata
and union territories. is represented inside an enclosure in a
3. The final three digits refer to individual square (chaukat).
post offices in the area within the sorting 2. Pithora is a ritual art form practiced
district. by the Bhil and Bhilala tribals of
Jharkhand.
How many of the above statements is/are
GS

incorrect? Which of the above statements is/ are


correct?
(a) Only one
(a) 1 only
(b) Only two
(b) 2 only
(c) All three
(c) Both 1 and 2
(d) None (d) Neither 1 nor 2

42. Consider the following statements regarding 45. Consider the following statements:
the Monuments of national importance in
1. The resolutions on Fundamental Rights
India:
and on abolition of salt monopoly of
1. The protection and upkeep of the the Government were adopted at the
monuments of national importance is Lucknow Session.
the responsibility of the Archaeological 2. The Congress was banned by the British
Survey of India (ASI). Government after the declaration of
2. Graves and cemeteries of British officers Purna Swaraj at Lahore Session.
and soldiers are not included in the list 3. In 1931, congress defined what Swaraj
of Monuments of National Importance would mean for the masses during
(MNI). Karachi session.

PTS2024/AIOMT/102023/07 10
Which of the statements given above is/are How many of the above given statements
correct? regarding the nuclear fusion is/are correct?
(a) 1 and 2 only (a) Only one
(b) 2 only (b) Only two
(c) 3 only (c) All three
(d) 2 and 3 only (d) None

46. With reference to Sangam literature, 50. Which of the following bacteria contains the
Tolkappiyam is the: Cry toxin, that makes the bacterium to be
used as a bio-control agent against insect
(a) Oldest surviving Tamil grammar
pests?
(b) One of the eight anthologies in
(a) Bacillus thuringiensis
Ettutokai
(b) Agrobacterium tumefaciens
(c) Bardic poetry of the Sangam era
(c) Azospirillum brasilense
(d) Prelude to Pattuppattu

RE
(d) Bacillus vulgaris
47. ‘Whitely commission’ was set by the British
government largely to address which of the 51. With reference to Pravasi Kaushal Vikas
following? Yojana (PKVY), consider the following
statements:
(a) Labour issues
1. National Skill Development Corporation
(b) Influence of Foreign Communist (NSDC) is the implementing agency of
Influence
(c) Giving voting rights to women
(d) Currency issues
O 2.
the PKVY.
Its aim to train and certify Indian
workforce who are keen on overseas
employment in selected sectors
SC
to facilitate overseas employment
48. Consider the following statements regarding opportunities.
Bhagavatism:
Which of the above statements is/are
1. It originated in post-Mauryan times and
incorrect?
centered around the worship of Vishnu.
2. Bhagavatism was marked by bhakti and (a) 1 only
ahimsa. (b) 2 only
GS

Which of the above statements is/are (c) Both 1 and 2


correct?
(d) Neither 1 nor 2
(a) 1 only
(b) 2 only 52. Under this scheme, there is a provision of
setting up of more than 14500 schools by
(c) Both 1 and 2
strengthening the existing schools from
(d) Neither 1 nor 2 amongst schools managed by Central
government/State/UT Government/local
49. Consider the following statements: bodies. These schools will showcase the
implementation of the National Education
1. The temperature requirement for Policy 2020. The duration of scheme is
nuclear fusion on earth is lower than on proposed to be from 2022-23 to 2026-27.
Sun due to lower gravity on the earth.
Identify the appropriate answer based on
2. Both Deuterium and Tritium nuclei fuse
information given above.
to form heavier nuclei in the nuclear
fusion. (a) NIPUN Bharat Mission
3. Tokamak is a chamber using a powerful (b) PM-SHRI scheme
magnetic field to form the heated
(c) PRAGYATA scheme
electrically-charged gas required for the
fusion reaction. (d) Kasturba Gandhi Balika Vidyalaya

11 PTS2024/AIOMT/102023/07
53. Which of the following are objectives of the 56. Consider the following statements about
SAMEEEKSHA initiative? Ocean currents:
1. Adoption of clean energy 1. Variation in density and gravity cause
2. Initiative to combat desertification in the movement of surface and deep-water
India currents.
3. To improve the energy efficiency of the 2. Cold currents are usually found on the
MSME sector east coast of the continents in the low
4. To maximise effectiveness and and middle latitude.
widespread adoption of energy efficient
Which of the given statements is/are
technologies
correct?
Select the correct answer using the codes
given below: (a) 1 only
(a) 1, 3 and 4 only (b) 2 only
(b) 1, 2 and 4 only (c) Both 1 and 2
(c) 1, 2 and 3 only (d) Neither 1 nor 2

RE
(d) 2, 3 and 4 only
57. Regarding climate variations across
54. The first national report on the state of different continents, consider the following
India’s tribal people’s health was submitted statements:
to the Government of India by the Expert 1. The Siberian type of climate is only
Committee on Tribal Health. In this regard, experienced in Northern hemisphere.
consider the following statements regarding
2. British type of climate region is
the findings of the report:
1.
O
The half of the total malaria deaths in
India occur among the tribal population 3.
influenced with a lot of cyclonic activity.
Central California, south-eastern
of the country. and southwestern Australia have
Mediterranean climate.
SC
2. The tribal Infant Mortality Rate (IMR)
during the period of 1988-2014 has
How many of the above statement is/are
doubled.
correct?
Which of the statements given above is/are
correct? (a) Only one

(a) 1 only (b) Only two


(b) 2 only (c) All three
GS

(c) Both 1 and 2 (d) None


(d) Neither 1 nor 2
58. Consider the following statements regarding
55. Consider the following statements: the physical features of India:
1. An overseas elector is a person who is a 1. The Himalayan Mountain range was
citizen of India, has acquired citizenship formed due to the collision of Eurasian
of any other country and is otherwise plate and the Indo-pacific plate.
eligible to be registered as a voter. 2. Northern plain formation took place
2. The Citizenship Act of 1955 prescribes because of Himalayan upliftment and
descent, naturalization, and subsidence of the northern flank of the
incorporation of territory as ways of peninsular plateau.
acquiring citizenship of India.
Which of the statements given above is/are
Which of the statements given above is/are
correct?
correct?
(a) 1 only (a) 1 only

(b) 2 only (b) 2 only


(c) Both 1 and 2 (c) Both 1 and 2
(d) Neither 1 nor 2 (d) Neither 1 nor 2

PTS2024/AIOMT/102023/07 12
59. Which of the following measures are 62. Consider the following:
categorized as ‘Above-the-Line’ fiscal
1. Kigali Amendment
measures?
1. Subsidized food programmes or health 2. Montreal Protocol
facilities 3. Paris Agreement
2. Capital Grants 4. Kyoto Protocol
3. Loans and Credit Guarantees
How many of the following international
4. Schemes like MGNREGA
treaties related to GHG emission are legally
5. Tax Cuts binding in nature?
6. Direct Transfers
(a) Only one
Select the correct option using the code given
below: (b) Only two

(a) 1, 2, 3, 4 and 5 only (c) Only three


(b) 1, 2, 3, 4, 5 and 6 (d) All four

RE
(c) 1, 2, 4, 5 and 6 only
(d) 1, 2, 3 and 4 only 63. Which of the following statement is
incorrect?
60. With reference to poverty in India, consider (a) Green Climate Fund (GCF) was
the following statements: established to reduce greenhouse gas
1. The percentage of poor in Scheduled (GHG) emissions in developed as well as

2.
castes.
O
Tribes is more than that of scheduled

Jharkhand has the highest proportion of


the population living below the national
developing countries.
(b) Global Environment Facility (GEF) is
a financial mechanism for UNFCCC and
SC
poverty line. Minamata convention.
Which of the statements given above is/are (c) States other than European Union can
correct? become the part European Environment
(a) 1 only Agency.
(b) 2 only (d) IPCC was established by the World
(c) Both 1 and 2 Meteorological Organization (WMO)
GS

and the United Nations Environment


(d) Neither 1 nor 2
Programme (UNEP).

61. If another global financial crisis happens


soon, how many among the following action/ 64. With reference to the characteristics of the
policies are most likely to give some immunity Black Carbon, consider the following:
to India?
1. Short-lived pollutant
1. Not depending on short-term foreign
borrowings 2. An aerosol

2. Opening more foreign banks 3. Reduction of surface albedo


3. Maintaining full capital account 4. Second-largest contributor to warming
convertibility
How many of the above-mentioned is/are
Select the correct answer using the code correct?
given below:
(a) Only one
(a) Only one
(b) Only two (b) Only two

(c) All three (c) Only three

(d) None (d) All four

13 PTS2024/AIOMT/102023/07
65. Consider the following pairs: Can India Make Progress?’ and also he
stressed on establishment and maintenance
Metal Associated Disease of an indigenous cultural identity. ‘Andher
poisoning Nagari’, a famous play of his, portrays a city
that falls into darkness due to the lack of
1. Lead - Itai-Itai sound administration signifying autocratic
British rule in India.
2. Methylmercury - Minamata Who among the following personalities is
described in the above passage?
3. Cadmium - Black foot disease
(a) Hem Chandra Bandyopadhyaya
4. Arsenic - Anaemia (b) Bharatendu Harishchandra

How many of the above pairs is/are correctly (c) Bankim Candra Chatterjee
matched? (d) Nabin Chandra Sen
(a) Only one
68. Which female social reformer wrote the short
(b) Only two stories: Sultana’s Dream and Avarodhbasini,

RE
(c) Only three criticizing the custom of purdah among
Muslims?
(d) All four
(a) Begum Hazrat Mahal
66. Gandhi and Tagore debated the social- (b) Abadi Bano Begum
political aspects of freedom. Which among (c) Bibi Amatus Salam
the following were ‘points of convergence’ in
their respective vision?
1.
O
Tagore backed the idea of fusion of
(d) Begam Rokey Sakhawat Hossain

elements and approaches of East and 69. The earliest available text from her literary
West in educational sphere while output was her Sanskrit poem “Lamentation
SC
Gandhi supported only Indian model of of Divine Language”, submitted to the
education. Oriental Conference at Berlin in 1881 and
highlighting the violence of colonialism. Her
2. Both Gandhi and Tagore believed that book “The High-Caste Hindu Woman” was
colonialism was more of an ‘internal based on the miseries of girls and women in
psychological construct’ rather than high-caste Hindu families. It is not wrong
external domination of Britishers. to say that she entered into the feminist
3. Gandhi believed in non-cooperation and discourse through her book Stri Dharma
GS

Satyagraha as political force but Tagore Niti (Morals for women). This book turned
believed in the Boycott and Atmashakti out to be a guide of women’s morality,
as political force. asking illiterate, ignorant women to recast
themselves in a more cultural mould through
4. Gandhi dismissed modernity as self-reliance and self-education. Her next
‘materialistic enslaving of humanity’ academic venture, Cry of Indian Women,
while Tagore admired modernity as more explicitly reflected her feminist thinking
‘growth in civilization’. and her desire to seek gender justice. She is:
Select the correct answer using the codes (a) Pandita Ramabai
given below:
(b) Mrs Annie Besant
(a) 1 and 2 only
(c) Tarabai Shinde
(b) 1 and 3 only
(d) Muthulakshmi Reddy
(c) 1, 2 and 4 only
(d) 2, 3 and 4 only 70. Consider the following statements:
1. Thorn forests occur in the areas that
67. He is known as the father of Hindi receive annual rainfall less than 50cm.
Literature. He founded and edited one of 2. Moist Deciduous Forests are found in
India’s first women’s journals, Bala Bodhini. the eastern slopes of Western Ghats and
In his 1884 address at Ballia entitled, ‘How Odisha.

PTS2024/AIOMT/102023/07 14
3. Swamp Forests are found along the 73. Which of the following statements is/are
Andaman and Nicobar Islands. incorrect?

How many of the above statements is/are 1. The publication of Hunter Commission
Report on the disturbances in Punjab
incorrect?
connected with the Rowlatt Satyagraha.
(a) Only one 2. The attainment of dominion status for
(b) Only two India was declared to be the immediate
goal of the Swaraj party.
(c) All three
Select the correct answer using the codes
(d) None below:
(a) 1 only
71. Consider the following statements:
(b) 2 only
1. A.O Hume and Annie Besant founded
(c) Both 1 and 2
the Theosophical Society in New York
who were inspired by Indian thought (d) Neither 1 nor 2

RE
and culture.
2. Pandita Ramabai set up Mahakali 74. Consider the following statements:
Pathshala in Calcutta to propagate 1. Pt. Nehru and Subhash Chandra Bose
female education without foreign moved a resolution to make poorna
influence. swaraj the goal of the Congress struggle
in December 1927.
3. Deoband Movement was established by
2. In the Nehru report, no special

Muslim community.
How many of the above statements is/are
O
Badr-ud-din Tyabji and Sir Syed Ahmad
Khan to train religious leaders for the
representation in the Central Parliament
was allowed except for the Muslims in
Bengal and the non-Muslims in the
North-West Frontier Province.
SC
incorrect? Which of the above statements is/are
(a) Only one correct?
(a) 1 only
(b) Only two
(b) 2 only
(c) All three
(c) Both 1 and 2
(d) None
GS

(d) Neither 1 nor 2


72. Which of the following statements is/are
correct? 75. With respect to ancient history of India,
consider the following statements:
1. Fort William and Haileybury College
1. Kanishka’s coins carry images of deities
were set up in India to train civil
from various religions.
servants.
2. Kushanas used local gold and silver
2. The Thuggee Act XXX of 1836 provides coins as high value currency.
the basis for the demarcation of ‘criminal
3. The Gupta dominions were organised
castes’ and ‘criminal tribes’ after the on the principle of decentralised
revolt of 1857. administration
Select the correct answer using the codes How many of the above statements is/are
below: correct?
(a) 1 only (a) Only one
(b) 2 only (b) Only two

(c) Both 1 and 2 (c) All three

(d) Neither 1 nor 2 (d) None

15 PTS2024/AIOMT/102023/07
76. Consider the following statements regarding Which of the statements given above is/are
poverty: correct?
1. Generational Poverty is a temporary (a) 1 only
type of poverty based on occurrence of
an adverse event like environmental (b) 2 only
disaster, job loss and severe health (c) Both 1 and 2
problem.
(d) Neither 1 nor 2
2. Situational Poverty is defined from the
social perspective such as living standard
compared to the economic standards of 79. Consider the following statements regarding
population living in surroundings. Isotherms and their nature in different
hemisphere:
3. Relative Poverty is handed over to
individual and families from one 1. Isotherms deviates less in southern
generation to the one. hemisphere than in northern hemisphere
and are almost parallel to latitudes.
4. Absolute poverty involves the scarcity of
basic food, clean water, health, shelter, 2. Isotherms are impacted by warm ocean

RE
education, and information. currents in both northern and southern
hemisphere.
How many of the above statements is/are
correct? Which of the statements given above is/are
correct?
(a) Only one
(b) Only two (a) 1 only

(c) Only three (b) 2 only

(d) All four


O (c) Both 1 and 2
(d) Neither 1 nor 2
77. Consider the following statements regarding
SC
Indian currency design mechanism: 80. Consider the following statements regarding
1. India’s currency notes printing presses SAMRIDH initiative:
at Nasik and Dewas are owned by the 1. It is a joint initiative of NITI Aayog and
RBI. World Health Organization (WHO).
2. The Coinage Act, 2011 gives the Union 2. It provides financial support to
Government the power to design and healthcare innovators.
mint coins in various denominations.
GS

Which among the above statements is/are


3. Indian currency notes are designed to
correct?
reflect the diverse cultural heritage of
the country. (a) 1 only
How many of the above statements is/are (b) 2 only
incorrect?
(c) Both 1 and 2
(a) Only one
(d) Neither 1 nor 2
(b) Only two
(c) All three 81. Consider the following statements regarding
(d) None simultaneous elections:
1. Simultaneous elections cannot be held
78. Consider the following statements regarding within the existing framework of the
‘Loess’: Constitution.
1. It is a yellow, friable fine dust blown 2. Simultaneous elections required
beyond the desert limit and is deposited replacing the ‘no-confidence motion’ with
on neighbouring lands. a ‘constructive vote of no-confidence.
2. It is a fine loam, rich in lime and 3. Simultaneous elections ensure timely
extremely porous. implementation of government policies.

PTS2024/AIOMT/102023/07 16
How many of the above statements is/are 2. Fundamental Rights under Article 19
correct? can be automatically suspended till the
end of the emergency.
(a) Only one
3. President can put the state assembly
(b) Only two
under suspension during president rule.
(c) All three
How many of the above statements is/are
(d) None correct?
(a) Only one
82. Consider the following statements regarding
(b) Only two
Comptroller and Auditor-General of India
(CAG): (c) All three
1. No minister can represent the CAG in (d) None
the parliament.
2. CAG control the issue of money from the 86. With reference to the Global Biofuels Alliance
consolidated fund of India. (GBA), consider the following statements:

RE
Which among the above statements is/are 1. It is an initiative to develop an
correct? alliance of governments, international
organisations and industry to facilitate
(a) 1 only
adoption of biofuels.
(b) 2 only
2. India is the largest Biofuel producer in
(c) Both 1 and 2 the world.
3. It was launched by India, at G20

83.
(d) Neither 1 nor 2

Which among the following subjects is


part of the XIth Schedule of the Indian
O 4.
Summit gaining consensus from World
Environment Facility (WEF).
The Alliance intends to standardize
certification for widespread adoption of
SC
Constitution?
biofuels.
(a) Land improvement, library, and cultural
activities. How many of the statements given above is/
are correct?
(b) Regulation of land-use and construction
of buildings. (a) Only one
(c) Vital statistics include registration of (b) Only two
births and deaths.
GS

(c) Only three


(d) Safeguarding the interests of weaker (d) All four
sections of society, including the
handicapped and mentally retarded.
87. Consider the following statements:

84. “Sentinel on the qui vive” phrase is relevant 1. G-20 Satellite Mission for Environment
in the context of which among the following? and Climate Observation aims to share
climate and weather data obtained from
(a) Supreme court of India Chandrayaan-3 Mission.
(b) Election commission of India 2. All the countries of Global South
(c) Comptroller and Auditor General of have agreed to join the G-20 Satellite
India Mission.

(d) Union public service commission Which of the statements given above is/are
correct?

85. Consider the following statements regarding (a) 1 only


effect of emergency in India: (b) 2 only
1. Only the term of the Lok Sabha can be (c) Both 1 and 2
extended during operation of emergency
and not for state assemblies. (d) Neither 1 nor 2

17 PTS2024/AIOMT/102023/07
88. Regarding Delhi Police establishment Act 2. It is commonly found in the lakes and
(DSPE), consider the following statements: ponds of central India.
1. Section 6A of the act provides that any 3. They are critically endangered as per
investigation into corruption charges the IUCN Red List.
against officials of the rank of Joint
4. They are omnivores.
Secretary and above can begin only after
the Central government’s approval. How many of the statements given above is/
2. In Vineet Narain (1997) case, the apex are correct?
court held that a statutory investigation (a) Only one
cannot be impeded by administrative
instructions. (b) Only two

3. The DSPE Act empowers the Central (c) Only three


Government to extend the jurisdiction of (d) All four
this special police force to other Union
Territories as well.
91. Consider the following statements:
How many of the statements given above is/
are incorrect? 1. As per the guidelines, the strategic

RE
installations of drones will differ from
(a) Only one state to state.
(b) Only two 2. The installations will include sites like
(c) All three airports, oil refineries, power plants,
and monuments.
(d) None
3. A Counter-Unmanned Aircraft

89. With reference to ‘Ayushman Bhava’


O
Programme, launched recently, consider the
System(C-UAS) will be established for a
set of legal procedures to handle rogue
drones.
following statements:
With reference to the National Counter
SC
Statement-I:
Rogue Drone Guidelines, how many of the
The Programme aims to redefine and shift statements given above is/are incorrect?
the healthcare accessibility from whole-of-
(a) Only one
nation and whole-of-society approach.
(b) Only two
Statement-I:
This campaign was initiated by the Ministry (c) All three
of Health and Family Welfare. (d) None
GS

Which one of the following is correct in


respect of the above statements? 92. Consider the following:
(a) Both Statement-I and Statement-II are 1. Increase in capital inflows into India
correct and Statement-II is the correct 2. Increase in foreign portfolio investments
explanation for Statement-II. (FPI)
(b) Both Statement-I and Statement-II 3. Tendency for the currency to depreciate
are correct and Statement-II is not the
correct explanation for Statement-I. 4. Investors to buy Indian government
bonds denominated in Rupee
(c) Statement-I is correct but Statement-II
is incorrect. 5. Increase Inflation

(d) Statement-I is incorrect but Statement- Which of the above given are probably the
II is correct. impacts when G-secs are traded in the bond
market?
90. Regarding the Peacock Soft-shelled Turtle (a) 1, 3, 4 and 5 only
(Nilssonia hurum), consider the following (b) 2, 4 and 5 only
statements:
(c) 1, 2, 3 and 4 only
1. They are freshwater turtles, native to
the Indian subcontinent. (d) 1, 2, 3, 4 and 5

PTS2024/AIOMT/102023/07 18
93. Consider the following statements regarding 96. Consider the following statements regarding
NASA’s OSIRIS-REx mission: ‘Pradhan Mantri Dakshta Aur Kushalta
1. OSIRIS-REx is a NASA spacecraft Sampann Hitgrahi Yojana’:
designed to study and collect samples 1. It is a National Action Plan for skilling
from an asteroid named Bennu. marginalized persons covering SCs,
OBCs, EBCs, DNTs, Sanitation workers
2. The primary objective of the OSIRIS-
including waste pickers.
REx mission is to return samples from
Bennu to Earth for further analysis. 2. National Scheduled Castes Finance and
Development Corporation (NSFDC) is
Which of the statements above is/are
solely responsible for the implementation
correct?
of the scheme.
(a) 1 only
Which of the statements given above is/are
(b) 2 only correct?
(c) Both 1 and 2 (a) 1 only
(d) Neither 1 nor 2 (b) 2 only

RE
(c) Both 1 and 2
94. Consider the following statements: (d) Neither 1 nor 2
1. Article 105 of the Constitution deals with
the powers of the Houses of Parliament 97. Which of the following best describes the
against a defamation suit. term “Global South”?

2. The protection under Article 105 does (a) It refers to countries located in the

Parliament.
O
not extend to the Attorney General of
India and non-members of the House of
southern hemisphere of the globe.
(b) It represents a group of countries that
have significant economic and political
influence in global affairs.
SC
3. The Supreme Court held that
parliamentarians are entitled to (c) It denotes nations that prioritize
immunity from criminal prosecution globalization and open-market policies.
when it comes to their speech and votes (d) It signifies a collective of developing
in the House. and emerging economies from various
How many of the statements given above are continents.
correct?
GS

98. With reference to Social Stock Exchange,


(a) Only one consider the following statements:
(b) Only two 1. They are the platforms where shares
(c) All three of companies after raising capital from
public investors are traded.
(d) None
2. Any not-for-profit organization (NPO)
working for social goals essentially
95. Which of the following is the primary focus of can raise capital from public investors
the Abraham Accords, a series of agreements by listing on SSE akin to what normal
signed in 2020? companies do for listing on stock
(a) Economic cooperation between Middle exchanges.
Eastern nations. Which of the statements given above is/are
(b) Resolution of the Israeli-Palestinian correct?
conflict. (a) 1 only
(c) Nuclear disarmament in the Middle (b) 2 only
East.
(c) Both 1 and 2
(d) Establishment of a regional security
alliance. (d) Neither 1 nor 2

19 PTS2024/AIOMT/102023/07
99. Consider the following statements regarding 100. Consider the following statements regarding
Pacific Decadal Oscillation (PDO): the United Nations Office on Drugs and
1. It is a sea surface temperature (SST) Crime (UNODC):
climate cycle (or teleconnection) 1. The United Nations Office on Drugs and
describing sea surface temperature Crime (UNODC) is the custodian of the
anomalies over the North-eastern Pacific
UNTOC.
Ocean.
2. The purpose of this Convention is to
2. The PDO oscillates between positive and
negative phases. promote cooperation to prevent and
combat transnational organized crime
3. Negative phase of the PDO tend to be
more effectively.
associated with periods of more rapid
global warming 3. India has so far not ratified the
4. It is a short-term climate pattern. convention.

How many of the above statements are How many of the statements given above are
correct? correct?

(a) Only one (a) Only one

RE
(b) Only two (b) Only two
(c) Only three (c) All three
(d) All four (d) None
O
™™™™™
SC
GS

PTS2024/AIOMT/102023/07 20
IAS Prelims - 2024
PTS2024/AIOMT/102023/07

ANSWER BOOKLET

ALL INDIA OPEN MOCK TEST


GENERAL STUDIES PAPER - I

Answer Key

Q. 1 (c) Q. 21 (a) Q. 41 (b) Q. 61 (a) Q. 81 (c)


Q. 2 (d) Q. 22 (a) Q. 42 (c) Q. 62 (d) Q. 82 (a)
Q. 3 (c) Q. 23 (c) Q. 43 (a) Q. 63 (a) Q. 83 (a)
Q. 4 (c) Q. 24 (c) Q. 44 (a) Q. 64 (d) Q. 84 (a)
Q. 5 (a) Q. 25 (c) Q. 45 (c) Q. 65 (a) Q. 85 (b)
Q. 6 (c) Q. 26 (b) Q. 46 (a) Q. 66 (c) Q. 86 (d)
Q. 7 (d) Q. 27 (a) Q. 47 (a) Q. 67 (b) Q. 87 (a)
Q. 8 (a) Q. 28 (c) Q. 48 (c) Q. 68 (d) Q. 88 (d)
Q. 9 (c) Q. 29 (c) Q. 49 (b) Q. 69 (a) Q. 89 (d)
Q. 10 (c) Q. 30 (d) Q. 50 (a) Q. 70 (d) Q. 90 (b)
Q. 11 (d) Q. 31 (c) Q. 51 (d) Q. 71 (c) Q. 91 (d)
Q. 12 (b) Q. 32 (a) Q. 52 (b) Q. 72 (b) Q. 92 (c)
Q. 13 (c) Q. 33 (d) Q. 53 (a) Q. 73 (d) Q. 93 (c)
Q. 14 (a) Q. 34 (d) Q. 54 (a) Q. 74 (c) Q. 94 (b)
Q. 15 (d) Q. 35 (a) Q. 55 (b) Q. 75 (b) Q. 95 (a)
Q. 16 (c) Q. 36 (d) Q. 56 (a) Q. 76 (a) Q. 96 (a)
Q. 17 (b) Q. 37 (b) Q. 57 (c) Q. 77 (a) Q. 97 (d)
Q. 18 (d) Q. 38 (d) Q. 58 (b) Q. 78 (c) Q. 98 (c)
Q. 19 (d) Q. 39 (a) Q. 59 (c) Q. 79 (c) Q. 99 (b)
Q. 20 (b) Q. 40 (b) Q. 60 (a) Q. 80 (b) Q. 100 (b)

PTS2024/AIOMT/102023/07
1. Correct Option: (c)  New 5G use cases will deliver new revenue
streams for communication service providers
Explanation: (CSPs) and new connectivity opportunities
 Statement 3 is incorrect: RSV can survive for subscribers. These use cases include:
for many hours on hard surfaces.  Cloud gaming
Supplementary notes:  AR/VR

Respiratory Syncytial Virus  Autonomous driving

 Respiratory syncytial virus, or RSV, is a  Fixed Wireless Access


common respiratory virus that usually  The future goal for ORAN is for any hardware
causes mild, cold-like symptoms. Most and software in the cellular network to
people recover in a week or two, but RSV interoperate both seamlessly and securely
can be serious, especially for infants and regardless of its originating vendor.
older adults.
 It is a negative-sense, single-stranded RNA 3. Correct Option: (c)
virus. Its name is derived from the large
cells known as syncytia that form when Explanation:
infected cells fuse.  All statements are correct.
 RSV can survive for many hours on hard
Supplementary notes:

RE
surfaces such as tables and crib rails. It
typically lives on soft surfaces such as Indian Biological Data Centre
tissues and hands for shorter amounts of
time.  The ‘Indian Biological Data Centre (IBDC)’
is the first national repository for life
 Like other respiratory viruses like science data in India. IBDC is mandated to
influenza (flu) or COVID-19, symptoms archive all life science data generated from
of RSV infection usually include runny publicly funded research in India. The data
O
nose, coughing, sneezing, fever, decreased center is supported by the Government of
appetite, and wheezing. The symptoms India (GOI) through the Department of
usually appear in stages. Biotechnology (DBT). It is being established
 An antiviral drug called palivizumab is at the Regional Centre of Biotechnology
available to prevent severe RSV illness in (RCB), Faridabad in the national capital
SC
high-risk infants (born prematurely or with region in collaboration with the National
congenital heart disease or chronic lung Informatics Centre (NIC), India. In times
disease). to come, IBDC is envisaged to emerge as
a major data repository for all life science
data emerging from India.
2. Correct Option: (d)  Indian Crop Phenome Database (ICPD) is a
Explanation: domain of the Indian Biological Data Center
(IBDC), Regional Centre for Biotechnology,
GS

 Both statements are correct.


Faridabad, developed for the digitization
Supplementary notes: of crop phenome data. ICPD would act as
single-stop user-friendly platform for freely
Open Radio Access Network (RAN) archiving, organizing, analyzing, and sharing
 The Radio Access Network (RAN) provides the multi-crop phenome data following
the critical technology to connect users, FAIR (Findable, Accessible, Interoperable
including mobile phones or enterprises, to and Re-usable) data principles.
the mobile network over radio waves. It
also acts as a bridge to access all the key
applications on the web.
4. Correct Option: (c)
 Current RAN technology is provided as a Explanation:
hardware and software integrated platform.  Statement II is incorrect: ACE2 is known
The ambition for Open RAN is to create a to be expressed both as membrane-bound
multi-supplier RAN solution that allows for receptors or as soluble proteins.
the separation - or disaggregation - between
hardware and software with open interfaces Supplementary notes:
and virtualisation, hosting software that
Angiotensin Converting Enzyme 2
controls and updates networks in the cloud.
(ACE2)
The promised benefits include supply chain
diversity, solution flexibility, and new  ACE2 is known to be expressed in various
capabilities leading to increased competition human organs, and its organ- and cell-
and further innovation. specific expression suggests that it may play

PTS2024/AIOMT/102023/07 2
a role in the regulation of cardiovascular stars will have different combinations of
and renal function, as well as fertility. brightness and temperature, but they all
ACE2 is an enzyme that can be found either obey the same relationship. For example,
attached to the membrane of cells (mACE2) smaller stars, like red dwarfs, will be
in the intestines, kidney, testis, gallbladder, relatively dim but also cool, with their
and heart or in a soluble form (sACE2). surfaces turning a characteristic shade of
 Both membranes bound and soluble ACE2 red. Medium stars, like the sun, will be both
are integral parts of the renin–angiotensin– hotter and brighter, turning white. The
aldosterone system (RAAS) that exists to largest stars will be both incredibly bright
keep the body’s blood pressure in check. and extremely hot, making them appear
While mACE2 does not appear to factor into blue.
the harmful phase of RAAS (the increase  All stars get progressively hotter and
of blood pressure), its existence is vital in brighter as they age, because the fusion of
order for the enzyme ADAM17 to cleave its hydrogen leaves behind helium. Once the
extracellular domain to create soluble ACE2 star runs out of available hydrogen in its
(sACE2). core, it turns to fusing helium — or even
 Soluble ACE2 lowers blood pressure by heavier elements, if the star is massive
catalyzing the hydrolysis of angiotensin II enough — at the end of its life. When this
(a vasoconstrictor peptide) into angiotensin happens, the star moves away from the
(1–7) (a vasodilator) which in turns binds main sequence, coming up with strange

RE
to MasR receptors creating localized new combinations of temperature and
vasodilation and hence decreasing blood brightness. For example, red giants are
pressure. This decrease in blood pressure incredibly bright but have relatively cool
makes the entire process a promising drug surfaces — something that can’t happen to
target for treating cardiovascular diseases. a hydrogen-burning main sequence star.
 mACE2 serves as the entry point into cells  Astronomers can use the main sequence
for some coronaviruses, including HCoV- to estimate the ages of stars, especially if
NL63, SARS-CoV, and SARS-CoV-2. The
O
SARS-CoV-2 spike protein itself is known to
those stars belong to a cluster. Stars within
a cluster tend to form around the same
damage the endothelium via downregulation time, which means that stars begin fusing
of ACE2. The human version of the enzyme hydrogen, and enter the main sequence, at
can be referred to as hACE2. roughly the same time.
SC
 ACE2 is a functional receptor for the spike  After enough time, the largest stars begin to
glycoprotein of the human coronavirus die, leaving the main sequence as they do.
HCoV-NL63 and the human severe acute As more time passes, smaller and smaller
respiratory syndrome coronaviruses, SARS- stars follow suit. Because all the stars in the
CoV and SARS-CoV-2, the latter is the cluster have the same age, astronomers can
causative agent of coronavirus disease-2019 use this observation to calculate that age.
(COVID-19).  But there is an exception for the blue
stragglers. Carrying out the first-ever
GS

comprehensive analysis of blue stragglers,


5. Correct Option: (a)
Indian researchers found that half of the
Explanation: blue stragglers in their sample are formed
through mass transfer from a close binary
 Option (a) is correct: Blue stragglers are
companion star, one third are likely formed
stars in a cluster that are very bright and
through collisions of 2 stars, and the
very blue but stubbornly persist on the main
remaining are formed through interactions
sequence. Unlike their more well-behaved
of more than 2 stars.
siblings, they burn hydrogen long after they
should have run out of fuel.  Initially, these blue stars still straggling
 They are only found in dense stellar above the turnoff were not considered as
systems, where distances between stars are part of these clusters. However, later studies
extremely small (a fraction of a light year). confirmed that these stars are indeed cluster
members, and they were termed “Blue
Supplementary notes: Stragglers”. The only probable way these
stars can still be present in these clusters is
Blue Straggler stars if they have somehow acquired extra mass
 All stars follow a particular path in life, along the way while on the main sequence.
known as the main sequence. The moment Confirming the mechanisms of the mass
they begin fusing hydrogen in their cores, gain required a study using a large sample
they maintain a strict relationship between of blue-straggler stars and estimates of the
their brightness and temperature. Different mass they have gained.

3
PTS2024/AIOMT/102023/07
6. Correct Option: (c)  All matter around us is made of elementary
particles, the building blocks of matter.
Explanation: These particles occur in two basic types
 Pair 4 is incorrectly matched: Gluons called quarks and leptons.
are the bosons that carry the strong nuclear  Each group consists of six particles, which
force. are related in pairs, or “generations”. The
lightest and most stable particles make up
Supplementary notes:
the first generation, whereas the heavier
Fundamental particles and less-stable particles belong to the
second and third generations.
 The standard model of particle physics, divides
every particle in the universe and even the  All stable matter in the universe is made
larger composite particles fit into two broad from particles that belong to the first
categories; fermions and bosons. generation; any heavier particles quickly
decay to more stable ones.
 Fermions such as quarks, electrons,
neutrinos, protons, and neutrons are  The six quarks are paired in three
the foundation of matter, while one generations – the “up quark” and the “down
category of bosons, the gauge bosons, are quark” form the first generation, followed
responsible for acting as the ‘carriers’ of at by the “charm quark” and “strange quark”,
least three of the four fundamental forces then the “top quark” and “bottom (or beauty)
— electromagnetism, the strong nuclear quark”. Quarks also come in three different

RE
force, and the weak nuclear force. That “colours” and only mix in such ways as to
means that fermions interact with each form colourless objects.
other via the exchange of gauge bosons.  Leptons come in two types: electrons,
 The most famous gauge boson is the photon, which have an electric charge of –1, and
the constituent particle of light and the neutrinos, which are electrically neutral.
mediator of the electromagnetic force. Leptons do not possess color charge and do
not interact via the strong force; this is the
O
 Gluons, the second discovered gauge boson,
main feature that distinguishes them from
are the bosons that carry the strong nuclear
quarks.
force. As a result, they are responsible for
‘sticking’ other particles together.  The six leptons are similarly arranged in
three generations – the “electron” and the
The W and Z bosons are the gauge bosons
SC

“electron neutrino”, the “muon” and the
responsible for carrying the weak nuclear
“muon neutrino”, and the “tau” and the “tau
force, stronger than gravity but only effective
neutrino”. The electron, the muon and the
across incredibly short ranges.
tau all have an electric charge and a sizeable
 The Higgs boson was first introduced into mass, whereas the neutrinos are electrically
the standard model of particle physics neutral and have very little mass.
to explain how the W and Z bosons got
their mass, but its mass-granting role as
the facilitator of the Higgs field was soon 7. Correct Option: (d)
GS

extended to almost all particles.


Explanation:
 There may also be bosons to carry the force
 All pairs are correctly matched.
of gravity, but that isn’t currently certain.
The gauge bosons are fundamental particles Supplementary notes:
— meaning they’re not comprised of smaller
particles — but there are other bosons that Polymers
are composed of smaller particles.

Polymer Monomer Uses


Rubber Isoprene (1, 2-methyl 1 – 1, 3-butadiene) Making tyres, elastic materials
BUNA – S (a) 1, 3-butadiene (b) Styrene Synthetic rubber
BUNA – N (a) 1, 3-butadiene (b) Vinyl Cyanide Synthetic rubber
Teflon Tetrafluoro Ethane Non-stick cookware – plastics
Terylene (a) Ethylene glycol (b) Terephthalic acid Fabric
Glyptal (a) Ethylene glycol (b) Phthalic acid Fabric
Bakelite (a) Phenol (b) Formaldehyde Plastic switches, Mugs, buckets

PTS2024/AIOMT/102023/07 4
PVC Vinyl Cyanide Tubes, Pipes
Melamine
(a) Melamine (b) Formaldehyde Ceramic plastic material
Formaldehyde Resin
Nylon-6 Caprolactam Fabric

8. Correct Option: (a) Supplementary notes:

Explanation: Sources of the Constitution at a Glance


 Option (a) is correct.
Sources Features Borrowed
Protists
Japanese Procedure established by Law
 Living things are divided into five kingdoms: Constitution
animal, plant, fungi, protist and monera.
French Republic and the ideals of liberty,
 Protists cannot be divided perfectly into Constitution equality, and fraternity in the
algae, protozoa, and fungi. As a result, Preamble.
the protists are spread across the major
conventional algal and fungal classifications Australian Concurrent List, freedom of trade,

RE
Constitution commerce and inter-course, and
(in kingdom systems) and the eukaryotes joint sitting of the two Houses of
generally (in the three-domain system). Parliament.

Supplementary notes: Russia Fundamental duties and the ideal


 Examples of protists include: Constitution of justice (social, economic and
political) in the Preamble.
 Amoebas (including nucleariids and
Foraminifera);
O Government
of India Act of
Federal Scheme, Office of
governor, Judiciary, Public
 Choanoflagellates; ciliates; 1935 Service Commissions, Emergency
provisions and administrative
 Diatoms; details.
SC
 Dinoflagellates;
British Parliamentary government, Rule
 Giardia; Constitution of Law, legislative procedure,
single citizenship, cabinet system,
 Oomycetes (including Phytophthora, the prerogative writs, parliamentary
cause of the Great Famine of Ireland); privileges and bicameralism.
and
US Constitution Fundamental rights, independence
 Plasmodium (which causes malaria); of judiciary, judicial review,
GS

impeachment of the president,


 Slime molds. removal of Supreme Court and
 A protist is any eukaryote that is not an high court judges and post of
vicepresident.
animal, (land) plant, or (true) fungus;
this definition excludes many unicellular Irish Directive Principles of State Policy,
Constitution nomination of members to Rajya
groups, like the Microsporidia (fungi), many
Sabha and method of election of
Chytridiomycetes (fungi), and yeasts (fungi), president.
and also a non-unicellular group included in
Canadian Federation with a strong Centre,
Protista in the past, the Myxozoa (animal).
Constitution vesting of residuary powers
in the Centre, appointment of
Note: Blue-green algae is a bacterium. state governors by the Centre,
and advisory jurisdiction of the
9. Correct Option: (c) Supreme Court.

Weimar Suspension of Fundamental Rights


Explanation: Constitution of during Emergency.
 Pair 2 is incorrectly matched: French Germany
Constitution - Ideals of liberty, equality,
South African Procedure for amendment of
and fraternity
Constitution the Constitution and election of
 Pair 4 is incorrectly matched: Russia members of Rajya Sabha.
Constitution - Fundamental Duties

5
PTS2024/AIOMT/102023/07
10. Correct Option: (c)  To make provision for just and humane
conditions of work and maternity relief
Explanation: (Article 42).
 Option (c) is correct: Indian constitution  To secure a living wage, a decent
secures a living wage, a decent standard of standard of life and social and cultural
life and social and cultural opportunities opportunities for all workers (Article
for all workers under Article 43 of DPSP 43).
(Socialistic Principles).
 To take steps to secure the participation
Supplementary notes: of workers in the management of
industries (Article 43 A).
Fixation of Floor Wage
 To raise the level of nutrition and the
 The Code on Wages, 2020 mentions the
standard of living of people and to
concept of a floor wage, which empowers
improve public health (Article 47).
the central government to fix floor wages
considering the minimum living standards
of workers. 11. Correct Option: (d)
 The floor wage is a baseline wage below Explanation:
which minimum wages cannot be fixed by
state governments.  Statement 1 is incorrect: The

RE
formation of new states, alteration of
 The Wage Code permits the fixation of
areas and boundaries can be amended by
different floor-level wages for different
a simple majority of the two Houses
geographical areas. However, this has given
of Parliament.
rise to the fear of flight of capital from areas
where the wage is higher to areas where the  Statement 2 is incorrect: Fundamental
wage is lower. Rights and Directive Principles of State
Policy can be amended by the special
O
Socialistic Principles majority of the Parliament.
 These principles reflect the ideology of Supplementary notes:
socialism. They lay down the framework of a
democratic socialist state, aim at providing Procedure for Amending the Indian
Constitution
SC
social and economic justice, and set the
path towards welfare state. They direct the  Article 368 in Part XX of the Constitution
state: deals with the power of parliament to amend
 To promote the welfare of the people by the constitution and its procedures.
securing a social order permeated by  It states that the Parliament may amend the
justice–social, economic and political Constitution by way of addition, variation or
and to minimise inequalities in income, repeal of any provision in accordance with
status, facilities and opportuni-ties4 the procedure laid down for the purpose.
GS

(Article 38).  However, the Parliament cannot amend


 To secure (a) the right to adequate those provisions which form the ‘basic
means of livelihood for all citizens; structure’ of the Constitution. This was ruled
(b) the equitable distribution of by the Supreme Court in the Kesavananda
material resources of the community Bharati case (1973).
for the common good; (c) prevention of  It provides for two types of amendments,
concentration of wealth and means of that is, by a special majority of Parliament
production; (d) equal pay for equal work and the special majority of parliament along
for men and women; (e) preservation of with the ratification of half of the State
the health and strength of workers and legislatures by a simple majority.
children against forcible abuse; and (f)
 Amendment of certain provisions of the
opportunities for healthy development of
Constitution requires amendment by a
children5 (Article 39). simple majority of each house present and
 To promote equal justice and to provide voting. These amendments are not deemed
free legal aid to the poor (Article 39 to be amendments under Article 368.
A). Simple Majority
 To secure the right to work, to  A number of provisions in the Constitution
education, and to public assistance can be amended by a simple majority of the
in cases of unemployment, old age, two Houses of Parliament outside the scope
sickness and disablement (Article 41). of Article 368. These provisions include:

PTS2024/AIOMT/102023/07 6
 Formation of new states and alteration Supplementary notes:
of areas, boundaries or names of existing
states, Office of Governor
 Abolition or creation of legislative  A governor holds office for a term of five
councils in states, years from the date on which he enters
upon his office. However, this term of
 Use of official language,
five years is subject to the pleasure of the
 Citizenship – acquisition, and President. Further, he can resign at any
termination, time by addressing a resignation letter to
 Elections to Parliament and state the President. The Supreme Court held
legislatures, that the pleasure of the President is not
 Fifth Schedule – administration of justifiable. The governor has no security of
scheduled areas and scheduled tribes, tenure and no fixed term of office. He may
be removed by the President at any time.
 Sixth Schedule – administration of tribal
areas.  The Constitution does not lay down any
grounds upon which a governor may be
Special Majority removed by the President. Hence, the
 Under Article 368(2), Parliament can amend National Front Government headed by
the Constitution by passing a Bill with a V.P. Singh (1989) asked all the governors

RE
special majority. to resign as they were appointed by the
 Fundamental Rights and Directive Congress government. Eventually, some
Principles of State Policy (DPSP) are the of the governors were replaced and some
two most important provisions that can be were allowed to continue. The same thing
amended by the special majority, but the was repeated in 1991, when the Congress
amendments should be within the Basic Government headed by P.V. Narasimha
structure of the constitution. Rao changed fourteen governors appointed

O
All provisions that do not require ratification by the V.P. Singh and Chandra Sekhar
by states, and those that come directly under governments.
the purview of Article 368, can be amended  The President may transfer a Governor
by the special majority. Special Majority appointed to one state to another state for
SC
with Consent of Half of States:
the rest of the term. Further, a Governor
 Those provisions of the Constitution whose term has expired may be reappointed
which are related to the federal structure in the same state or any other state. A
of the polity can only be amended by a governor can hold office beyond his term
special majority of the Parliament and of five years until his successor assumes
with the consent of half of the state charge. The underlying idea is that there
legislatures by a simple majority.
must be a governor in the state and there
Important provisions that require
GS

 cannot be an interregnum.
ratification by the states include the
 The President can make such provision
election of President, Supreme Court and
High Courts, representation of states in as he thinks fit for the discharge of the
Parliament, distribution of legislative functions of the governor in any contingency
powers between the Union and the not provided for in the Constitution, for
states, and the extent of executive power example, the death of a sitting governor.
of the Union and the states. Thus, the chief justice of the concerned state
high court may be appointed temporarily to
 Most importantly, an amendment to
discharge the functions of the governor of
Article 368 itself, requires ratification by
the states. that state.
 A decision to remove a Governor can be
challenged in a court of law. In such cases,
12. Correct Option: (b) first the petitioner will have to make a
Explanation: prima facie case of arbitrariness or bad
 Statement 1 is incorrect: The governor faith on part of the central government. If
has no security of tenure and no fixed term a prima facie case is established, the court
of office. can require the central government to
produce the materials on the basis of which
 Statement 3 is incorrect: A governor
the decision was made in order to verify the
whose term has expired may be reappointed
presence of compelling reasons.
in the same state or any other state.

7
PTS2024/AIOMT/102023/07
13. Correct Option: (c) chairman or the speaker should be informed
by the concerned authority, of the reason for
Explanation: the arrest. But a member can be arrested
 All statements are correct. outside the limits of the house on criminal
charges against him under the Preventive
Supplementary notes: Detention act, the Essential Services
Parliamentary Privileges Maintenance Act (ESMA), the National
Security Act (NSA), or any such act.
 Parliamentary privileges are special rights,
 Right to Prohibit the Publication
immunities and exemptions enjoyed by the
of Proceedings: Article 105(2) of the
two Houses of Parliament, their committees
Constitution, no person shall be held liable
and their members. These privileges
for publishing any reports, discussions etc.
are defined in Article 105 of the Indian
of the house under the authority of the
Constitution.
member of the house. For paramount and
 Under these privileges, the members of national importance, it is essential that the
Parliament are exempted from any civil proceedings should be communicated to the
liability (but not criminal liability) for any public to aware them of what is going on in
statement made or act done in the course the parliament.
of their duties. The privileges are claimed
 Right to Exclude Strangers: The
only when the person is a member of the
members of the house have the power and

RE
house. As soon as s/he ends to be a member,
right to exclude strangers who are not
the privileges are said to be called off.
members of the house from the proceedings.
Parliament has not made any special law to
This right is very essential for securing free
exhaustively codify all the privileges. They
and fair discussion in the house.
are rather based on five sources:
 Constitutional provisions
14. Correct Option: (a)
 Various laws made by Parliament
O
 Rules of both the Houses Explanation:
 Parliamentary conventions  Statement 2 is incorrect: Appointment of
permanent judges in a high court having
 Judicial interpretations jurisdiction over more than one state the
SC
Privileges: Chief Justice of that High Court would
initiate proposal to the Governor of the
 Freedom of Speech in Parliament: The concerned states.
freedom of speech and expression guaranteed
to a citizen under Article 19(2) is different Supplementary notes:
from the freedom of speech and expression
Appointment of High Court Judges
provided to a member of the parliament. It
has been guaranteed under Article 105(1) of  The Chief Justice and Judges of the
the Indian constitution. But the freedom is High Courts are to be appointed by the
GS

subject to rules and orders which regulate President under clause (1) of Article 217
the proceedings of the parliament. of the Constitution. The Judges of the
Jammu & Kashmir High Court are to
 Limitations: Freedom of speech should
be appointed by the President under
be in accordance with the constitutional
section 95 of the Constitution of Jammu &
provisions and subject to rules and Kashmir. Appointments to the High Court
procedures of the parliament, as stated should be made on a time bound schedule
under Article 118 of the Constitution. so that the appointments are made well
Under Article 121 of the Constitution, the in advance preferably a month before the
members of the parliament are restricted occurrence of the anticipated vacancy.
from discussing the conduct of the judges of
the Supreme Court and the High Court.  When a permanent vacancy is expected to
arise in any year in the office of a Judge, the
 Freedom from Arrest: The members Chief Justice will as early as possible but at
enjoy freedom from arrest in any civil case least 6 months before the date of occurrence
40 days before and after the adjournment of the vacancy, communicate to the Chief
of the house and also when the house is Minister of the State his views as to the
in session. No member can be arrested persons to be selected for appointment.
from the limits of the parliament without Full details of the persons recommended,
the permission of the house to which s/he in the format given in Annexure-I should
belongs so that there is no hindrance in invariably be sent. Before forwarding his
performing their duties. If the detention of recommendation, the Chief Justice must
any members of the parliament is made, the consult two of his senior-most colleagues

PTS2024/AIOMT/102023/07 8
on the Bench regarding the suitability of  It shall be lawful for the President by order
the names proposed. All consultation must to establish such a Council, and to define
be in writing and these opinions must be the nature of the duties to be performed by
sent to the Chief Minister along with the it and its organization and procedure.”
recommendations.  The Commission on Centre-State Relations
Appointment of permanent judges in a under the Chairmanship of Justice R. S.
high court having jurisdiction over more Sarkaria in its report in January 1988
than one state: recommended that:
 For appointments in these High Courts,  A permanent Inter-State Council called
the Chief Justice would initiate proposal the Inter-Governmental Council (IGC)
to the Governor of the State where the seat should be set up under Article 263.
of High Court is situated, and in the case  The IGC should be charged with the
of High Court of Punjab & Haryana, to the duties set out in clauses (b) and (c) of
senior of the two Governors of these States, Article 263, other than socio-economic
who would do the coordination and obtain planning and development.”(para 9.10.01
the views of other Governor and Chief of the Report)
Ministers concerned in writing and forward
the same along with the recommendations  Government of India accepted the
of the Chief Justice of the High Court to recommendation of the Sarkaria Commission
the Union Minister of Law, Justice and to set-up an Inter-State Council and notified

RE
Company Affairs for further appropriate the establishment of the Inter-State Council
action. vide Presidential Order dated 28.05.1990.

 In case, any of the State authorities wishes


to recommend a name different from the one 16. Correct Option: (c)
recommended by the Chief Justice of the
High Court, he should send the same to the Explanation:
Chief Justice of the High Court concerned  Option (c) is correct.
O
for his consideration. The initiation of a
recommendation for filling up of a vacancy
 Laterite Soil: The term is derived from
the word ‘Later’ which means ‘brick’.
would be made only by the Chief Justice of
Laterite soil is found in those regions of the
the High Court concerned.
country which receive heavy rainfall with
SC
an alternate dry and wet period - mainly,
15. Correct Option: (d) near the coasts. This kind of soil becomes
soft when wet and hardens when dry. In
Explanation: these climatic conditions, leaching of soil
 Option (d) is correct: The public interests takes place, which is a process in which the
would be served by the establishment of a fertile portion of the soil gets washed away
Council. by heavy rains. They are formed from the
decomposition of rocks and contain iron
Supplementary notes: oxide, which gives them red or pink color.
GS

Inter-State Council This type of soil is ordinarily deficient in


nitrogen and is weak in lime content; it is
 Article 263 of the Constitution of India acidic soil. It is found in several parts of the
provides for the establishment of an Inter- country mainly Western and Eastern Ghats,
State Council. Vindhyas, Malwa plateau, and Satpuras.
 “263. Provisions with respect to an inter- The states where this type of soil can be
State Council – If at any time it appears found are West Bengal, Andhra Pradesh,
to the President that the public interests Bihar, Meghalaya, Assam, and Odisha, to
would be served by the establishment of a name a few. Laterite soil supports crops like
Council charged with the duty of: Rice, Ragi, Sugarcane, rubber, coconut, tea,
 inquiring into and advising upon disputes coffee and Cashew nuts.
which may have arisen between States;
Supplementary notes:
 investigating and discussing subjects in
which some or all of the States, or the Soil types and their locations
Union and one or more of the States,  Black Soil: This type of soil is made up of
have a common interest; or volcanic rocks and lava. Black soil is also
 making recommendations upon known as ‘regur’ which is derived from the
any such subject and, in particular, Telugu word ‘reguda’. Black soil is also
recommendations for the better co- known as Black Cotton Soil as cotton is an
ordination of policy and action with important crop that is grown in this type of
respect to that subject, soil. This soil is rich in calcium carbonate,

9
PTS2024/AIOMT/102023/07
potash, lime, and magnesium carbonate but  Plates move horizontally over the
has poor phosphorus content. It is mostly asthenosphere as rigid units.
found in areas such as Gujarat, Madhya  A plate may be referred to as the continental
Pradesh, and Maharashtra. It is also found plate or oceanic plate depending on which of
in states like Tamil Nadu, Andhra Pradesh, the two occupy a larger portion of the plate.
and Karnataka. Black soil is excellent
 Pacific plate is largely an oceanic plate
and clayey and can hold a lot of moisture.
whereas the Eurasian plate may be called a
It becomes sticky in the rainy season and
continental plate.
develops cracks when dry. Black soil is
good for producing cotton, oilseeds, wheat,  The major plates are as follows : (i) Antarctica
linseed, millets, and tobacco. and the surrounding oceanic plate (ii)
North American (with western Atlantic
 Red Soil: This type of soil is formed as a floor separated from the South American
result of weathering of metamorphic and plate along the Caribbean islands) plate
igneous rocks. The red colour of the soil (iii) South American (with western Atlantic
comes from the high percentage of iron floor separated from the North American
content. The soil’s texture varies from plate along the Caribbean islands) plate
sandy to clayey, but it is mainly loamy. It is (iv) Pacific plate (v) India-Australia-New
rich in potash content but lacks phosphate, Zealand plate (vi) Africa with the eastern
hummus, and nitrogen content. The red soil Atlantic floor plate (vii) Eurasia and the
is found in regions such as Tamil Nadu, adjacent oceanic plate.

RE
Madhya Pradesh, Jharkhand, Odisha,
 Minor plates - Cocos plate : Between
some parts of Karnataka, and southeast
Central America and Pacific plate (ii) Nazca
Maharashtra. plate : Between South America and Pacific
 Peat Soil: The accumulation of a high plate (iii) Arabian plate : Mostly the Saudi
number of organic matter in the soil in Arabian landmass (iv) Philippine plate :
humid regions results in the formation of Between the Asiatic and Pacific plate (v)
peaty soils. These types of soils constitute Caroline plate : Between the Philippine and
Indian plate (North of New Guinea) (vi) Fuji
O
about 10 to 40% of the organic matter and
also a reasonable number of soluble salts. plate : North-east of Australia.
Peaty soils are heavy, black, and have high New plates map
acidic content. They are low in phosphate
and potash content. Peaty and marshy soils  The new model for tectonic plates better
SC
are found in a few districts of Kerala. On explains the spatial distribution of 90
the other hand, marshy soils are found in percent of earthquakes and 80 percent
coastal areas of some states such as Tamil of volcanoes from the past two million
Nadu, Bihar, Almora district of Uttaranchal, years.
and Sundarbans of West Bengal.  Existing models only capture 65 percent of
earthquakes.
The map includes several new microplates
17. Correct Option: (b) 
to the existing tectonic plate model like the
GS

Explanation:  Macquarie microplate which sits


 Pair 1 is incorrect: Cocos plate - Between south of Tasmania; and
Central America and Pacific plate  The Capricorn microplate which
 Pair 2 is correct: Nazca plate - Between separates the Indian and Australian
South America and Pacific plate plates.

 Pair 3 is incorrect: Capricorn microplate  Researchers found that plate boundary


- Between Indian and Australian plates zones account for nearly 16% of the Earth’s
crust and an even higher proportion, 27%,
 Pair 4 is correct: Caroline plate - Between of continents.
the Philippine and Indian plate
 The new model now includes all the
Supplementary notes: deformation zones north of India as the
plate bulldozes its way into Eurasia.
Tectonic plates
A tectonic plate (also called lithospheric

plate) is a massive, irregularly-shaped slab
18. Correct Option: (d)
of solid rock, generally composed of both Explanation:
continental and oceanic lithosphere.
 Statement 1 is incorrect: The western
 The lithosphere includes the crust and top cyclonic disturbances experienced in the
mantle with its thickness range varying north and north-western parts of the
between 5-100 km in oceanic parts and country are brought in by the subtropical
about 200 km in the continental areas. westerly jet stream.

PTS2024/AIOMT/102023/07 10
 Statement 2 is incorrect: The sub-tropical  An easterly jet stream, called the sub-
easterly jet stream blows over Peninsular tropical easterly jet stream blows over
India. peninsular India, approximately over 14°N
 Statement 3 is incorrect: Western during the summer months.
disturbances are extra-tropical cyclones  Jet streams are a narrow belt of high
developed in the Mediterranean Sea. altitude (above 12,000 m) westerly winds in
the troposphere.
Supplementary notes:
 Their speed varies from about 110 km/h in
Western Cyclonic Disturbances summer to about 184 km/h in winter.
 The western cyclonic disturbances are  A number of separate jet streams have been
weather phenomena of the winter months identified. The most constant are the mid-
brought in by the westerly flow from the latitude and the sub tropical jet stream.
Mediterranean region.
 They usually influence the weather of the
north and north-western regions of India.
19. Correct Option: (d)
 A characteristic feature of the cold weather Explanation:
season over the northern plains is the inflow  All pairs are correctly matched
of cyclonic disturbances from the west and
the northwest. 1. Machchhu river - Gujarat

RE
 These low-pressure systems, originate 2. River Lukha - Meghalaya
over the Mediterranean Sea and western
3. River Bhogdoi - Assam
Asia and move into India, along with the
westerly flow. 4. Parbati river - Madhya Pradesh
 They cause the much-needed winter Supplementary notes:
rains over the plains and snowfall in the
mountains. Rivers in India
O
Although the total amount of winter rainfall
 Machchhu river
locally known as ‘mahawat’ is small, they are
of immense importance for the cultivation  The Machchhu river originates from Madla
of ‘rabi’ crops. hills in the Surendranagar district of
SC
Gujarat and ends 141.75 km downstream in
 The peninsular region does not have a well- the Rann of Kutch.
defined cold season.
 Tributaries: Beti, Asoi, Jamburi, Benia,
 There is hardly any noticeable seasonal Machchhori, Maha are the tributaries of
change in temperature pattern during Machchhu river.
winters due to the moderating influence of
the sea. River Lukha
Jet Streams  One of Meghalaya’s major rivers is the
GS

Lukha. The river has been rendered dead


 In summer, a low-pressure area develops by unchecked mining activity and effluents
over interior Asia as well as over released by cement factories.
northwestern India.
 The upper air circulation in this region is
River Bhogdoi
dominated by a westerly flow. An important  The rampant coal mining in Nagaland,
component of this flow is the jet stream. coupled with waste discharge from tea
These jet streams are located approximately estates and encroachment have been
over 27°-30° north latitude, therefore, they sounding death knells for River Bhogdoi in
are known as subtropical westerly jet Assam.
streams.
Parbati River
 Over India, these jet streams blow south of
the Himalayas, all through the year except  Parbati River is a river in Madhya Pradesh
in summer. and Rajasthan, India that flows into the
Chambal River.
 The western cyclonic disturbances
experienced in the north and north-western International Day of Action for Rivers
parts of the country are brought in by this (IDAR)
westerly flow.  Every year on March 14, the International
 In summer, the subtropical westerly jet Day of Action for Rivers (IDAR) is observed
stream moves north of the Himalayas with to promote awareness of the importance of
the apparent movement of the sun. the river systems on the planet earth.

11
PTS2024/AIOMT/102023/07
 This year, the 25th anniversary of the day Supplementary notes:
was observed.
Mineral and metal deposits
 The day was earlier known as International
Day against Dams, for Rivers, Water, and Bauxite
Life.  Bauxite is the ore which is used in
 Theme: Rights of Rivers. manufacturing of aluminium. Bauxite is
found mainly in tertiary deposits and is
 It calls for the designation of rivers as a associated with laterite rocks occurring
national treasure. extensively either on the plateau or hill
ranges of peninsular India and also in the
coastal tracts of the country.
20. Correct Option: (b)
 Odisha happens to be the largest producer
Explanation: of Bauxite. Kalahandi and Sambalpur are
the leading producers.
 Option (b) is correct.
 Odisha is India’s largest bauxite
Supplementary notes: producer accounting for about 49% of
the total production followed by Gujarat
Indian Monsoon
(24%), Jharkhand (9%), Chhattisgarh and
 The monsoons are experienced in the tropical Maharashtra (8% each).
area roughly between 20° N and 20° S. To
Iron Ore

RE
understand the mechanism of the monsoons,
the following facts are important.  India is endowed with fairly abundant
resources of iron ore. It has the largest
 The differential heating and cooling of
reserve of iron ore in Asia.
land and water creates low pressure on
the landmass of India while the seas  The two main types of ore found in our
around experience comparatively high country are haematite and magnetite.
pressure.  About 95 per cent of total reserves of iron
ore is located in the States of Odisha,
O
 The shift of the position of Inter Tropical
Convergence Zone (ITCZ) in summer, Jharkhand, Chhattisgarh, Karnataka, Goa,
Telangana, Andhra Pradesh and Tamil
over the Ganga plain (this is the
Nadu.
equatorial trough normally positioned
about 5°N of the equator. It is also  The iron ore mines occur in close proximity
SC
known as the monsoon trough during to the coal fields in the north-eastern
the monsoon season). plateau region of the country which adds to
their advantage.
 The presence of the high-pressure area,
east of Madagascar, approximately at Graphite
20°S over the Indian Ocean. The intensity  Graphite, also known as plumbago or
and position of this high-pressure area blacklead or mineral carbon, is a stable
affects the Indian Monsoon. form of naturally occurring carbon.
The Tibetan plateau gets intensely Both flaky and amorphous varieties of
GS

 
heated during summer, which results graphite are produced in India. The quality
in strong vertical air currents and the of graphite depends upon its physical
formation of low pressure over the qualities and carbon content.
plateau at about 9 km above sea level.  Odisha was the leading producing State
 The movement of the westerly jet stream to contributing 59% to the total output during
the north of the Himalayas and the presence 2018-19, followed by Jharkhand (40%) and
of the tropical easterly jet stream over the Kerala (1%).
Indian peninsula during summer.  Active mining centres of graphite are in
Palamu district in Jharkhand; Nuapada &
Balangir districts in Odisha; and Madurai
21. Correct Option: (a) & Sivaganga districts in Tamil Nadu.
Explanation:  In Odisha, areas in and around Balangir
are the chief mining centres where several
 Statement 2 is incorrect: The two main graphite grades are produced.
types of ore found in our country are
Hematite and Magnetite (NOT limonite). Lithium
 Limonite are inferior ores.  The Geological Survey of India estimates it
 These are found in Damuda series in has discovered 5.9 million tons of lithium
Raniganj coal field, Garhwal in Uttarákhand, resources in the mountainous Salal-
Mirzapur in Uttar Pradesh and Kangra Haimana area of the Reasi district in the
valley of Himachal Pradesh. union territory of Jammu and Kashmir

PTS2024/AIOMT/102023/07 12
22. Correct Option: (a) Council decides to take up this issue, they
are well within their interest to take it up
Explanation: and discuss.
 Option (a) is correct.
Supplementary notes: 24. Correct Option: (c)
Indian Peninsula Explanation:
 The Peninsular plateau is a tableland  Option (c) is correct.
composed of the old crystalline, igneous and
metamorphic rocks.  Disposal of fixed assets is taken away from
the total while calculating Gross Fixed
 It was formed due to the breaking and Capital Formation (GFCF).
drifting of the Gondwana land and thus,
making it a part of the oldest landmass.  Gross fixed capital formation, abbreviated
as GFCF, consists of resident producers’
 The plateau has broad and shallow valleys investments, deducting disposals, in fixed
and rounded hills. This plateau consists of
assets during a given period.
two broad divisions, namely, the Central
Highlands and the Deccan Plateau.  It also includes certain additions to the
 The part of the peninsular plateau lying to value of non-produced assets realized by
the north of the Narmada River, covering a producers or institutional units.
major area of the Malwa plateau, is known Supplementary notes:
as the Central Highlands.

RE
 The Vindhyan range is bounded by the Gross Fixed Capital Formation (GFCF)
Satpura range on the south and the Aravalis  Gross capital formation refers to the
on the northwest. ‘aggregate of gross additions to fixed assets
 The further westward extension gradually (that is fixed capital formation) plus change
merges with the sandy and rocky desert of in stocks of inventories’ during the counting
Rajasthan. period.
Fixed asset refers to the construction,
23. Correct Option: (c)
O 
machinery and equipment. Gross fixed
capital formation (GFCF) is defined
Explanation: as the acquisition of produced assets
 Option (c) is correct: Petrol and diesel (including purchases of second-hand
assets), including the production of
SC
taxes are not under the ambit of good and
services yet. such assets by producers for their own use.

Supplementary notes:  Construction for military purposes


(other than construction or alteration of
Taxes applied on petrol/diesel by Central family dwellings for military personnel),
and State governments acquisition of defence equipment, durable
 The central government currently earns an goods in the hands of the households
excise duty of Rs. 32.90 per litre of petrol and increase in the stocks of defence
sold and Rs. 31.80 per litre of diesel sold. materials are excluded from the scope of
GS

gross fixed capital formation. Gross fixed


 Excise duty on fuel makes up about 18.4% of
capital formation (GFCF) includes the
the Centre’s gross tax revenues. Petroleum
acquisition of produced assets including
and alcohol, on an average, account for 25-
35% of states’ own tax revenue, as per the purchases of second-hand assets.
RBI’s Study of Budgets 2020-21.  It also includes the production of such
 Value-added tax or a sales tax is a tax on assets by producers for their own use,
sale of commodities. It is a state level sales minus disposals. However, the expenditure
tax. The tax rate is imposed as a percentage incurred on usual/ routine repair and
of value added. Hence, it is called VAT. After maintenance is not covered for compilation
the introduction of GST only few products of capital formation.
like petrol and diesel are under VAT. The  Gross Fixed Capital Formation (GFCF)
state governments charge a value-added measures the value of acquisitions of new
tax or a sales tax on every litre of petrol and
or existing fixed assets by the business
diesel sold. The rate varies across states.
sector, governments and “pure” households
 States apply an ad valorem VAT or sales (excluding their unincorporated enterprises)
tax on the base price, freight charges, excise fewer disposals of fixed assets.
duty and dealer commission on petrol and
diesel. Therefore, state collections also rise  GFCF is a component of the expenditure,
as the Centre hikes excise duties. and thus shows something about how much
of the new value- added in the economy is
 As fuel prices have surged in the recent
invested rather than consumed. One of the
past, so has the talk around bringing petrol
reasons for low GFCF is due to low spending
and diesel under the ambit of goods and
services tax (GST). Whenever the GST by the private sector in the capital asset.

13
PTS2024/AIOMT/102023/07
 The growth of the productive capacity 26. Correct Option: (b)
of the economy of the State depends on
its rate of capital accumulation and it Explanation:
is assessed by estimating the Capital  Statement 1 is incorrect: A Giffen good
Formation of that State. is a rare type of good, where an increase in
 The higher the rate of growth of Capital price causes an increase in demand.
Formation, the higher would be the Supplementary notes:
productive capacity of the economy,
whereas its paucity leads to the low level Types of Goods in Indian Economy
of production at a higher cost.  Giffen Good: A Giffen good is a low income,
 Thus, Capital Formation serves as a non-luxury product that defies standard
very important indicator in measuring economic and consumer demand theory. A
the magnitude of growth of productive rare type of good, where an increase in price
potential of the economy. causes an increase in demand. The reason is
that the income effect of a rise in the price
causes you to buy more of this cheap good
25. Correct Option: (c) because you can’t afford more expensive
goods.
Explanation:
 Inferior Good: It is one whose demand drops

RE
 Both statements are correct. when people’s incomes rise. The demands for
Supplementary notes: inferior goods move in the opposite direction
of the income of the consumer. Examples
Participatory Notes of inferior goods include low quality food
 FIIs must register with the Securities and items like coarse cereals. As the income of
Exchange Board of India. the consumer increases, the demand for
an inferior good fall, and as the income
O
 Participatory notes allow non-registered decreases, the demand for an inferior good
investors to invest in the Indian market. rise. This is because when the income of the
 P-Notes are financial instruments consumer increases, he will substitute these
required by investors or hedge funds goods with superior goods like wheat and
rice. However, if the income of the consumer
SC
to invest in Indian securities without
having to register with the Securities declines, he will spend substantial part of
and Exchange Board of India (SEBI). his income on this good.
 A hedge fund is a limited partnership of  Veblen Good: It is a good for which
private investors whose money is managed demand increases as the price increases.
by professional fund managers who use Veblen goods are typically high-quality
a wide range of strategies, including goods that are made well, are exclusive, and
leveraging or trading of non-traditional are a status symbol. Examples of Veblen
GS

assets, to earn above-average investment goods include designer jewellery, yachts,


returns. and luxury cars.

 Investments flowing through P-notes


are considered as offshore derivative 27. Correct Option: (a)
investments (ODIs). Indian securities
Explanation:
market regulator, SEBI issued the new
Regulations for Foreign Portfolio Investors,  Option (a) is correct.
participatory notes where it got formally Supplementary notes:
defined under the tag “Offshore Derivative
Instrument” (ODIs) in Section 2(1)(j) of Foreign investment in India
the said regulation.  Foreign investment in India is regulated
 The P-notes holder does not enjoy any under codified foreign exchange regulations,
voting rights in relation to security/shares sector specific policies/regulations,
referenced. Also, the investor in P-notes government policies as well as international
does not own the underlying Indian agreements.
security, which is held by the FII who issues  Primarily, foreign investment is regulated
the Participatory Notes. Thus, the investors through the Foreign Exchange Management
in Participatory Notes derive the economic Act, 1999 as amended from time to time
benefits of investing in the security without (FEMA) and rules/ regulations issued
holding it. thereunder.

PTS2024/AIOMT/102023/07 14
 The main objective of FEMA is to regulate, that crypto exchanges earn income in the
consolidate and amend the law relating form of transaction fees for completion of a
to foreign exchange to facilitate foreign transaction effectively, the GST authorities
investment, external trade and payments would treat the same as commission income
and promote the orderly development and which would be taxable at 18%.
maintenance of foreign exchange market in  Recently, it was all over the news that the
India. Directorate General of GST Intelligence
 INDIRECT FOREIGN INVESTMENT (DGGI) conducted search and raids on
– has been defined in Regulation 14(1) WazirX, Coin switch Kuber, Coin DCX
(v) of FEMA Notification No. 20 as under: (being some of the biggest crypto exchanges
“‘Indirect foreign investment’ means entire of India), and other such crypto exchanges
investment in other Indian companies by over potential GST evasion. Therefore,
an Indian company (IC), having foreign crypto exchanges which want to enter the
investment in it provided; Indian market should be mindful of such
requirements.
 Indian Company is not ‘owned and
controlled’ by resident Indian citizens  No such regulation exists in India which
and/or Indian Companies which are prohibits mining of crypto assets.
owned and controlled by resident Indian
citizens or
29. Correct Option: (c)
Where the Indian company is owned or

RE

controlled by non-residents. Explanation:
 Downstream investment is investment  Statement 2 is incorrect: The WPI
made by an Indian entity which has total comprises of three major groups:
foreign investment in it or an Investment  Primary Articles (eg- Food Articles,
Vehicle in the capital instruments or the Vegetables, Milk, Minerals etc.) has the
capital of another Indian entity. weightage of 22.62%.
If the investor company has total foreign

O
investment in it and is not owned and not
 Fuel and Power (eg- LPG, Petrol etc) has
the weightage of 13.15%.
controlled by resident Indian citizens or
is owned or controlled by persons resident  Manufactured Products (e.g.-
outside India then such investment shall manufacture of food products, sugar,
manufacture of textiles etc.) has the
SC
be “Indirect Foreign Investment” for the
weightage of 64.23%.
investee company.
 The major components in CPI (C) are as
 Asset reconstruction Company can access
follows (along with their weights):
100% funding in FDI from automatic route.
 Food and Beverages – 45.86%

28. Correct Option: (c)  Housing – 10.07%


 Fuel and Light – 6.84%
Explanation:
GS

 Clothing and Footwear – 6.53%


 Option (c) is correct.
 Pan, tobacco and intoxicants – 2.38%
Supplementary notes:  Miscellaneous – 28.32%
Regulation on crypto assets in India Supplementary notes:
 At the outset, currently, there is no specific
law/regulation/licensing requirement that Wholesale Price Index (WPI) and
is applicable on the crypto exchanges/ Consumer Price Index (CPI)
businesses which are operating in India.  Services are not traded/transacted in the
 Depending on the form of entity, the crypto wholesale markets. So, WPI data does not
exchanges would be required to obtain include the inflation due to services. The
customary approvals from the applicable CPI basket consists of services like housing,
authorities for doing business in India. education, medical care, recreation etc.
which are not part of WPI basket.
 For instance, if a crypto exchange is
operating in the form of a company then  WPI provides estimates of inflation at the
it shall have to adhere to the necessary wholesale transactions level for the economy.
provisions of the Companies Act, 2013, and This helps in timely intervention by the
Government to check inflation, in particular
its underlying rules
inflation in essential commodities, before
 Goods and Services Tax (GST) is an indirect the price increase spills over to retail prices.
tax in India which typically applies on The CPI (C) is used as nominal anchor for
supply of goods and services. Considering conduct of monetary policy in India.

15
PTS2024/AIOMT/102023/07
 Monetary Policy Committee is mandated 31. Correct Option: (c)
to keep CPI (C) in range 2% – 6%. So CPI
is used for inflation targeting. Explanation:
 CPI is also used as deflators in the  Statement 4 is incorrect: Easing/
National Accounts. improving the supply of an item in high
demand and short supply to ease down its
 CPI is also used for calculating Dearness
price by importing, etc. is a Supply Side
Allowance.
measure to combat inflation.
 CPI includes indirect taxes. So, when
government increases GST rate, it is captured Supplementary notes:
in the CPI data. But in the new series of Demand-Pull Inflation
WPI (2011-12), government has excluded
indirect taxes while measuring WPI. This is  By reducing the money, the government
in consonance with international practices is pumping into the economy, especially
and will make the new WPI conceptually consumptive, non-capital type, like salaries,
closer to Producer Price Index (PPI). HRA, DA, pensions, tax free incomes, etc. the
government reduces the disposable income
of households, thus curbing the pressure of
30. Correct Option: (d) demand from these economic agents, thus
reducing inflation from the demand side.
Explanation:

RE
 When the Central Banks use a Dear Money
 Statement 1 is incorrect: An anti-
policy - that is - increasing key policy rates
dumping duty is a protectionist tariff that
like Repo Rate, CRR, SLR, etc., the banks
a domestic government imposes on foreign
are obliged to retain more reserves, as well
imports that it believes are priced below fair
as discouraged to lend out more, due to the
market value.
high rates they must pay. This reduces
 Statement 2 is incorrect: Countervailing money supply in the system and individuals
duty (CVD) is a specific form of duty that
O
and businesses are unable to demand more,
the government imposes in order to protect thus easing demand side inflation.
domestic producers by countering the
 Higher interest rates on loans reduce the
negative impact of import subsidies.
demand in related sectors of the economy,
Supplementary notes: thus easing the demand pressure which was
SC
causing inflation. For example, increasing
Anti-dumping duty & countervailing the rates of home loans can help curb the
duty inflation in prices of bricks, mortar etc.
 World Trade Organization (WTO) deals  Easing/ improving the supply of an item in
with the rules of trade between nations–also high demand and short supply to ease down
operate a set of international trade rules, its price by importing, etc. is a Supply Side
including the anti-dumping duties. It also measure to combat inflation. For example,
permits the imposition of countervailing
GS

when the price of pulses were skyrocketing


duty by its member countries. in 2017 due to low domestic production, the
 An anti-dumping duty is a protectionist government imported a record amount of
tariff that a domestic government imposes pulses to stabilize the prices of this essential
on foreign imports that it believes are priced food item.
below fair market value.
 Dumping is a process wherein a company 32. Correct Option: (a)
exports a product at a price that is
significantly lower than the price it normally Explanation:
charges in its home (or its domestic) market.  Statement 1 is correct: The Northern
While the intention of anti-dumping duties Montane forests include the Himalayan
is to save domestic jobs, these tariffs can moist temperate forests, Himalayan dry
also lead to higher prices for domestic temperate forests, and Alpine forests. The
consumers. Himalayan Moist Temperate Forests have
 Countervailing duty (CVD) is a specific mixed species of broad-leafed evergreen
form of duty that the government imposes trees and conifers.
in order to protect domestic producers by  Statement 2 is incorrect: The boreal
countering the negative impact of import forests are the earth’s northernmost and
subsidies. largest land biome. Boreal landscapes are
 CVD is thus an import tax by the importing characterized by a low diversity of tree
country on imported products. species.

PTS2024/AIOMT/102023/07 16
 Statement 3 is correct: The deltas of the  Dense mangroves are the common varieties
Ganga, the Mahanadi, the Krishna, the with roots of the plants submerged under
Godavari and the Kaveri are covered by water. The deltas of the Ganga, the
mangrove vegetation. Mahanadi, the Krishna, the Godavari and
Supplementary notes: the Kaveri are covered by such vegetation.
 In the Ganga-Brahmaputra delta, Sundari
Forest Types and Locations
trees are found, which provide durable hard
 The boreal forest (or “taiga”) is the world’s timber. Palm, coconut, Keori, agar, etc., also
largest land biome. grow in some parts of the delta.
 Boreal
 forests, or taiga, are the Earth’s 33. Correct Option: (d)
northernmost forests.
 The boreal ecozone principally spans 8 Explanation:
countries: Canada, China, Finland, Japan,  Option (d) is correct: The ice-albedo
Norway, Russia, Sweden and the United feedback, lapse rate feedback, water vapour
States. feedback (Change in Water Vapour amplify
 It is typically comprised of coniferous tree or weaken temperature range) and ocean
species such as pine, spruce and fir with heat transport are the primary causes.
some broadleaf species such as poplar and

RE
birch. Supplementary notes:
 Most boreal landscapes are characterized Arctic amplification
by a low diversity of tree species, of which  Polar amplification happens when changes
gymnosperms such as Abies, Larix, Pinus, and to the earth’s atmosphere led to a larger
Picea species usually dominate, with varying
difference in temperature near the north
proportions of angiosperm Populus, Betula,
and south poles than to the rest of the
and Alnus species.
world.

O
This low tree diversity belies the thousands
 This phenomenon is measured against
of species of living organisms that thrive
within boreal stands. the average temperature change of the
planet.
 Boreal forests are also an important carbon
SC
sink. Like all forests they absorb carbon  These changes are more pronounced at
dioxide –a main contributor to global the northern latitudes and are known as
warming and climate change—removing it the Arctic amplification.
from the atmosphere and helping to keep  It occurs when the atmosphere’s net
the entire planet healthy.
radiation balance is affected by an increase
Montane Forests in India in greenhouse gases.
 Montane forests grow in mountainous Causes the Arctic Amplifications
regions. As the temperature decreases with
GS

an increase in height, there is a change  The ice-albedo feedback, lapse rate feedback,
in the cover of the natural vegetation at water vapour feedback (Change in Water
altitude. Montane forests can be divided Vapour amplify or weaken temperature
into two types—Northern Montane forests range) and ocean heat transport are the
and Southern Montane forests. primary causes.
 The Northern Montane forests include  Sea ice and snow have high albedo (measure
the Himalayan moist temperate forests, of reflectivity of the surface), implying that
Himalayan dry temperate forests, and they are capable of reflecting most of the
Alpine forests. solar radiation as opposed to water and
 They are found in regions that receive land.
rainfall between 100 cm and 300 cm. The  As the sea ice melts, the Arctic Ocean will be
temperature ranges between 12°C and more capable of absorbing solar radiation,
13°C. thereby driving the amplification.
 These are found in the Himalayan zone  The lapse rate or the rate at which the
from Kashmir to Sikkim and Arunachal
temperature drops with elevation decreases
Pradesh.
with warming.
Mangrove Forests  Studies show that the ice-albedo feedback
 The mangrove tidal forests are found in the and the lapse rate feedback are responsible
areas of coasts influenced by tides. Mud and for 40% and 15% of polar amplification
silt get accumutated on such coasts. respectively.

17
PTS2024/AIOMT/102023/07
34. Correct Option: (d) 35. Correct Option: (a)
Explanation: Explanation:
 Statement 2 is incorrect: Some CMEs  Statement 1 is incorrect: It is a freshwater
exhibit signatures at the solar surface and lake in Odhisha.
in the lower corona as the eruption occurs.  Statement 2 is correct: The lake is an
Thus, enabling their prediction before important habitat for vulnerable species
arriving at near-Earth satellites. However, such as Cyprinus carpio and river tern.
a significant fraction of CMEs exhibits no
such detectable signatures and are known Supplementary notes:
as “stealth CMEs”.
Ramsar wetlands
Supplementary notes:  Tampara lake is among the most prominent
Geomagnetic storms freshwater lakes in Odisha, situated in
Ganjam district.
 Geomagnetic storms are disturbances to
Earth’s magnetic field caused by solar  The wetland is an important habitat for
material from coronal mass ejections (CME) vulnerable species such as Cyprinus carpio,
— large expulsions of plasma and magnetic common pochard (Aythya ferina), and river
field from the sun’s atmosphere. tern (Sterna aurantia).

RE
 Recently, India has added 11 more Ramsar
 Coronal mass ejections (CMEs) – among
sites, or wetlands that are of international
the most energetic events originating from
importance, taking the number of such sites
the Sun – can cause significant and sudden
to 75.
disruption to the magnetic and particulate
environment of the heliosphere.  Tamil Nadu has maximum no. of Ramsar
sites (14), followed by UP which has 10 nos.
 The solar wind conditions that are
of Ramsar sites.
effective for creating geomagnetic storms
O
are sustained (for several to many hours)
periods of high-speed solar wind, and most 36. Correct Option: (d)
importantly, a southward directed solar
wind magnetic field (opposite the direction Explanation:
SC
of Earth’s field) at the dayside of the  Option (d) is correct.
magnetosphere.
Supplementary notes:
 Some CMEs exhibit signatures at the
solar surface and in the lower corona as About terms of ecology
the eruption occurs, thus enabling their  Carrying capacity
prediction before arriving at near-Earth
satellites.  The maximum number of individuals
in a single population that a given
However, a significant fraction of CMEs
GS

 environment can sustain at a given


exhibit no such detectable signatures and time.
are known as “stealth CMEs”.
 The carrying capacity is a characteristic
 The NOAA Geomagnetic Storm Scale of a single population of a single species
indicates the severity of geomagnetic at a single time in a single place.
storms. It is denoted by a G followed by a
number from 1 to 5, with 1 being a minor  Biomagnification
event, and 5 being an extreme event.  The process by which some substances
 The disturbance storm time (Dst) index, has increase in concentration in a food
been used historically to characterize the chain or food web.
size of a geomagnetic storm.  Biomagnification occurs because certain
substances, including some pesticides
 In addition, there are currents produced in
and heavy metals, are not easily degraded
the magnetosphere that follow the magnetic
and can accumulate in organisms’ tissues
field, called field-aligned currents, and these
or internal organs.
connect to intense currents in the auroral
ionosphere. These auroral currents, called  Eutrophication
the auroral electrojets, also produce large  The buildup of nitrogen and
magnetic disturbances. phosphorous in an aquatic environment.
 In space, a ring of westward current around Eutrophication usually leads to an
Earth produces magnetic disturbances on exponential growth in bacteria and
the ground. algae.

PTS2024/AIOMT/102023/07 18
 Niche  Climax Community:
 An organism’s role in an environment,  As succession progresses, the
including how it uses its resources, community of organisms undergoes a
relates to other organisms, and times its series of changes. Over time, the species
reproduction. composition becomes relatively stable
and reaches a state of equilibrium
 Each individual organism has a niche in its known as the climax community. This
population, community, and ecosystem, but community is well-adapted to the
niches are flexible and change depending on prevailing environmental conditions
circumstances. and tends to persist until another major
disturbance occurs.
37. Correct Option: (b)
38. Correct Option: (d)
Explanation:
 Option (b) is correct.
Explanation:
 Option (d) is incorrect: Endemism is the
Ecological condition of being endemic, or restricted
Types of Seres in geographical distribution to an area or
Succession
region. This does not show species richness.
Succession initiating on sandy
Psammosere

RE
areas. Supplementary notes:

Succession starting in saline Endemism


Halosere
soil or water.  Endemism is the condition of being endemic
or restricted in geographical distribution to
Succession of microorganism an area or region.
Senile
on dead matter.
 While endemism and being endangered
Development of vegetation in or threatened are different things, being
Eosere
an era.
O endemic to a small area is often a warning
sign that a species may become threatened
Supplementary notes: or endangered.
Ecological Succession  Endemism refers to the presence of a
SC
species only in a particular region and not
 Ecological succession is a process by which anywhere else. For example, the Lion-tailed
an ecosystem undergoes predictable and macaque which is endemic to the Western
orderly changes in structure and composition Ghats of India.
over time. This occurs in response to various
disturbances or environmental changes.
39. Correct Option: (a)
 Primary vs. Secondary Succession:
Primary succession occurs in an entirely Explanation:
GS


new or barren environment, such as a  Both Statements are correct.
volcanic island or a recently formed sand
Supplementary notes:
dune. It starts from scratch, with no soil
or living organisms. Thermal pollution
 Secondary succession, on the other  Thermal pollution is defined as a sudden
hand, occurs in an area that has been increase or decrease in the temperature
disturbed but still retains some soil and of a natural body of water, which may be
seed bank. Examples include abandoned an ocean, lake, river, or pond, by human
agricultural fields, areas affected by influence.
wildfires, or landslides.  This normally occurs when a plant or facility
 Pioneer Species: takes in water from a natural resource and
puts it back at an altered temperature.
 In both primary and secondary succession, Usually, these facilities use it as a cooling
pioneer species are the first organisms method for their machinery or to help better
to colonize the newly available habitat. produce their products.
These species are often hardy and can
 When a power plant first opens or shuts
tolerate harsh conditions like extreme
down for repair or other causes, fish and
temperatures, low nutrient levels, or
other organisms adapted to temperature
high salinity. They play a crucial role range can be killed by the abrupt change
in creating conditions suitable for other, in water temperature, either an increase or
less hardy species to thrive. decrease, known as “thermal shock”.

19
PTS2024/AIOMT/102023/07
40. Correct Option: (b) Code in India. It is a code in the post office
number of the postal code system which
Explanation: is used in India Post for segregating the
 Statement 2 is incorrect: Its excavation mails.
includes the stone portrait of Mauryan  The PIN code consists of six digits. The postal
Emperor Ashoka, surrounded by his queens address coding system was introduced by
and female attendants, with the inscription
Shriram Bhikaji in the year 1972. He was
“Raya Asoko” in Brahmi on it.
the then additional secretary in the Union
Supplementary notes: Ministry of Communications.
Kanaganahalli  PIN code is a six-digit long code used by the
Indian Postal Department. Each digit of a
 Kanaganahalli was a small and ordinary
pin code has a specific meaning.
village on the bank of Bhima in Karnataka.
 The first digit represents the region in India.
 A Maha Stupa has been discovered
here which was referred to as Adholoka The second digit is the sub-region, while the
Maha Chaitya (the Great Stupa of the third digit is the sorting district. The last
netherworlds) in the inscriptions. three digits represent the post office within
the district.
 The Maha Stupa is believed to have been
developed in three constructional phases  There are nine PIN regions in India, out of
– Maurya, Early Satavahana and Later which eight belong to geographical regions,

RE
Satavahana periods stretching from 3rd while the digit 9 is used for the army Postal
Century B.C. to 3rd Century A.D. The Service.
Stupa is believed to have been destroyed in
an earthquake.
 The other recoveries included:
42. Correct Option: (c)
 Around 60 dome slabs with the sculptural Explanation:
rendering of Jataka stories; Both statements are correct.
O

 Portrait of Ashoka;
Supplementary notes:
 Shatavahana monarchs and certain
unique depictions of Buddhist Monuments of national importance in
missionaries sent by Ashoka to different India
SC
parts;
 The Economic Advisory Council (EAC) to
 72 drum-slabs decorated with a variety the Prime Minister has released a report
of Dharma-Chakras, Stupas, the first titled ‘Monuments of National Importance:
sermon, Bodhi-tree, Naga Muchulinda, The Urgent Need for Rationalization’.
Vihara complexes;
 India currently has 3,693 monuments
 Over 10 inscribed sculptures of the of national importance (MNI) and their
Buddha, over a dozen Buddha-Padas;
protection and upkeep is the responsibility
Fragments of Ayaka pillars, umbrella
GS

 of the Archaeological Survey of India (ASI).


stones and shafts, parts of sculptures of
Yakshas and lion; and  The major problem plaguing the identification
and preservation of monuments of national
 250 Brahmi inscriptions with varied importance lies in the Ancient Monuments
palaeographical features. and Archaeological Sites and Remains
(AMASR) Act,1958.
41. Correct Option: (b)  Neither the Act nor the National Policy for
Conservation (2014) have defined what the
Explanation:
term ‘national importance’ means.
 Statement 1 is incorrect: The six-digit
pin code used by India Post to deliver mails  The Act also does not have a substantive
was created on August 15, 1972 by Shriram process/ criteria prescribed for identifying
Bhikaji Velankar. a monument as a monument of national
importance. In absence of well-defined
 Statement 2 is incorrect: The first digit of
principles, the selection of monuments of
the six-digit postal code indicates the zone –
national importance seems to be arbitrary
North, east, west or south.
 With 506 monuments under its belt,
Supplementary notes: Karnataka continues to top the list in
India’s Postal Code South India, followed by Tamil Nadu (413).
Nationally too, Karnataka is in the second
 The term Postal Index Number (PIN) is
place after Uttar Pradesh, which has 745
popularly known as PIN code/Postal
such monuments.

PTS2024/AIOMT/102023/07 20
 Large number of MNI seems not to have  A geographical indication right facilitates
national importance or historical or cultural those who have the right to use the
significance. The report estimates that indication to prohibit its usage by a third
around a quarter of the current list of 3,695 party whose product does not conform to the
MNI may not have ‘national importance’ applicable standards. However, a protected
per se. GI does not permit the holder to forbid
 For instance, around 75 graves and someone from making a product using the
cemeteries of British officers and same approaches as those set out in the
soldiers that have neither architectural standards for that indication.
significance nor historical or cultural
importance is on the list.  A registered GI shall be valid for 10 years
and can be renewed on payment of renewal
fee.
43. Correct Option: (a)
Explanation: 44. Correct Option: (a)
 Statement 1 is incorrect: The Geographical
Explanation:
Indications of Goods (Registration and
Protection) Act, 1999 seeks to provide for  Statement 2 is incorrect: Pithora is a
the registration and better protection of ritual art form practiced by the Bhil and
geographical indications relating to goods Bhilala tribes of the Western Madhya

RE
in India. Pradesh area spilling into Gujarat Western
 Statement 2 is incorrect: This tag is valid plateau.
for a period of 10 years following which it  This is a ritual performance that required
can be renewed. collectiveparticipation of more than four
Supplementary notes: to five people, but at present most Pithora
artists draw it individually and the art is
GI Tag recognized as an individual divine git given

O
A GI or Geographical Indication is a name or to the artist.
a sign given to certain products that relate
to a specific geographical location or origins Supplementary notes:
like a region, town, or country. Warli Painting
SC
 Using Geographical Indications may be
 The themes of these paintings are generally
regarded as a certification that the product
inspired from daily life, nature, epics and
is produced as per traditional methods, has
certain specific qualities, or has a particular legends.
reputation because of its geographical origin.  In this painting, the goddess of trees and
Geographical indications are typically used plants - Palaghata, who symbolises fertility
for spirit drinks, foodstuffs, agricultural and abundance, is represented inside
products, handicrafts, and industrial an enclosure in a square (chaukat) and
GS

products. accompanied by multitudes of line drawings


 GI Tag ensures that none other than those of people engaged in hunting, dancing or
registered as authorized users are allowed cultivating the land.
to use the popular product name. In order
 The great Warli folk painting is a form of
to function as a GI, a sign must identify a
product as originating in each place. tribal art mainly created by tribal people
in the northern region of the Sahyadri
 Geographical Indications are covered as a Range, which comprises cities like Jawhar,
component of intellectual property rights
Palghar, Dahanu, Talasari, Mokhada, and
(IPRs) under the Paris Convention for
Vikramgarh of Palghar district, in India.
the Protection of Industrial Property. At
the International level, GI is governed by  This warli tribal painting art form first
the World Trade Organisation’s (WTO’s) originated in Maharashtra, which is still
Agreement on Trade-Related Aspects of practised frequently. It is believed to be one
Intellectual Property Rights (TRIPS). of the oldest kinds of art forms in history.
 In India, Geographical Indications  It is pretty popular in India and has a
registration is administered by the distinct charm everywhere it is expressed,
Geographical Indications of Goods but it has not gained much recognition
(Registration and Protection) Act, 1999 outside its territory.
which came into force with effect from 15th
September 2003. The first product in India  Warli art is a folk art of Maharashtra is
to be accorded with GI tag was Darjeeling a set of basic geometric shapes: a circle, a
tea in the year 2004-05. triangle, and a square.

21
PTS2024/AIOMT/102023/07
Pithora Painting  The Karachi Resolution was to remain, in
 The Pithora paintings are done by the essence, the basic political and economic
programme of the Congress in later years.
Rathwa, Bhil, Nayak and Tadi tribes of
Gujarat, parts of Rajasthan and Madhya  Viceroy Irwin in fact had signed Gandhi
Pradesh. Irwin pact (Delhi Pact) on an equal footing
with the Congress and thus raised the
 The origin of these paintings goes back a
prestige of Congress. But a marked change
few thousand years, arising from the cave
was noticed in the Government attitude
paintings at a local hilltop near koraj-i-
after Gandhi returned from 2nd Round
dungar. These paintings speak of the social,
Table Conference.
cultural and mythological life and beliefs of
the tribals.  On January 4, 1932, Gandhi was arrested,
Congress organisations at all levels were
 The Pithora wall paintings are done on the
declared to be illegal soon after.
main wall of the house, which divides the
verandah from the kitchen. This part of the
house is considered sacred to Pithoro, the 46. Correct Option: (a)
god of food grains and the principal deity of
the tribe. Explanation:
 The same wall is used by the Warli and the  Option (a) is correct
Saora tribes for some of their paintings.
Supplementary notes:

RE
The Pithora paintings are also made if
the children are unwell, or if the children Tolkappiyam
or cattle are unyielding. These are seldom  The ancient Tamil of the Sangam poems is
made for decorative or ornamental purposes different from modern Tamil.
and mostly depict the legends of creation.
 The Tolkappiyam is the oldest surviving
 They also paint all three walls of the Tamil grammar; parts of it go back to the
verandah with a variety of scenes such early centuries CE.
O
as other deities, ghosts and ancestors.
Particularly interesting is the depiction  Such grammatical texts tell us about the
of sexual intercourse between men and structure of ancient languages and they
women, as well as animals. also contain incidental historical references
to their time
SC
 The Sangam corpus includes six of the
45. Correct Option: (c) eight anthologies of poems included in the
Explanation: Ettutokai (The Eight Collections), and nine
of the ten pattus (songs) of the Pattuppattu
 Statement 1 is incorrect: Two resolutions (The Ten Songs).
were adopted one on Fundamental Rights
and the other on National Economic  The style and certain historical references in
Programme which made the session the poems suggest that they were composed
particularly memorable. The Delhi Pact or between the 3rd century BCE and the
GS

Gandhi-Irwin Pact was endorsed and the 3rd century CE. They were compiled into
goal of purna Swaraj was reiterated. anthologies in about the mid-8th century.

 Statement 2 is incorrect: There was no  A few centuries later, these anthologies


such ban on the Congress in the first phase were collected into the super-anthologies
of CDM. (i.e., anthologies of anthologies) called the
Ettutokai and the Pattuppattu.
Supplementary notes:  The earliest parts of the first two books of
Karachi Session of INC (1931) the Tolkappiyam can also be included in
Sangam literature.
 In March 1931, a special session of the
Congress was held at Karachi to endorse  The Tolkappiyam is essentially a work on
the Gandhi-Irwin Pact. grammar, but it also includes a discussion of
phonology, semantics, syntax, and literary
 Resolutions passed at Karachi while conventions.
disapproving of and dissociating itself from
political violence, the Congress admired the  There are two kinds of Sangam poems—
‘bravery and ‘sacrifice’ of the three martyrs. akam and puram. Akam poems had love
as their theme, while puram poems were
 It was the first time the Congress spelt out mostly about war. A. K. Ramanujan (1999)
what Swaraj would mean for the masses- “in describes puram poetry as ‘public poetry’
order to end exploitation of masses, political which dealt with all kinds of themes other
freedom must include economic freedom of than love, such as good and evil, community
starving millions.” and kingdom.

PTS2024/AIOMT/102023/07 22
47. Correct Option: (a) 49. Correct Option: (b)
Explanation: Explanation:
 Option (a) is correct: The Royal  Statement 1 is incorrect: On the Sun,
Commission on Labour in 1931 pointed out due to the strong gravity, hydrogen atoms
the need for systematic collection of labour fuse at 15 million °C. On Earth, because of
statistics. the weaker gravitational forces, they need
 It observed that the policy must be built to be heated at temperatures as high as 150
on facts as the uncertainty of facts would million °C in order to collide.
lead to confusion and conflict regarding
Supplementary notes:
its aim.
Nuclear Fusion
Supplementary notes:
 Fusion reaction is a nuclear process by
Whitely commission which nuclei of two light elements fuse to
 The Royal Commission on Labour (or produce a fast, heavier nucleus and an even
the Whitley Commission on Labour) faster nucleon, i.e., a neutron or a proton.
was a Royal commission set up in 1929 There is a small mass difference, say m,
to investigate the working conditions between the initial and the final reaction
on plantations in India. products which gets converted into energy
The report surprised many by concurring through Einstein’s equation E=mc2, c being

RE

with the criticisms of Mahatma Gandhi and the speed of light.
others that poverty was the cause of India’s  For such a reaction to occur, the reacting
social and industrial problems. nuclei need to have enough kinetic energy to
 It was also critical of British employers’ role overcome the repulsive electrostatic barrier
in perpetuating the problems. between any two of them.
 A Deuterium and a Tritium nucleus fuse to
produce a Helium nucleus and a neutron.
48. Correct Option: (c)
O The reaction produces 17.6 MeV of energy,
Explanation: out of which the Helium carries 3.5 MeV
and the neutron 14.1 MeV. In a plasma
 Both statement are correct:
undergoing fusion, the reactions can be self-
SC
Supplementary notes: sustained, as part of the kinetic energy of
the resulting charged Helium can be used
Bhagavatism to maintain the very high temperatures
 Bhagavatism originated in post-Maurya required to sustain the fusion reactions.
times and centred on the worship of Vishnu  In the core of the Sun, hydrogen atoms move
or Bhagavata. at incredible speed. Light atoms of hydrogen
 Vishnu was a minor god in Vedic times. He fuse into one heavier atom of helium. The
represented the sun and also the fertility reaction releases lots of energy in the form
GS

cult. By the second century BC he was of light and heat. On the Sun, due to the
merged with a god called Narayana, and strong gravity, hydrogen atoms fuse at 15
came to be known as Narayana-Vishnu. million°C. On Earth, however, because of
 Bhagavatism was marked by bhakti and the weaker gravitational forces, they need
ahimsa. to be heated at temperatures as high as 150
million °C in order to collide.
 Bhakti meant the offer of loving devotion.
It was a kind of loyalty offered by a tribal  For decades scientists have been trying to
to his chief or by a subject to his king. figure out how to produce this energy through
various experiments. Although the principle
 Ahimsa, or the doctrine of non-killing of
is simple, they face several challenges. At
animals, suited the agricultural society
and was in keeping with the old cult 150 million °C, hydrogen atoms crush and
of life- giving fertility associated with end up forming an ‘electrically-charged
Vishnu. gas’ known as plasma. They came up with
the idea of a Tokamak: a chamber using a
 The new religion was sufficiently liberal powerful magnetic field to contain the hot
to attract foreigners. It also appealed plasma.
to artisans and merchants who became
important under the Satavahanas and  In a tokamak, the plasma has a shape like
Kushans. an inflated tube of a car wheel, enclosed in a
magnetic cage. The magnetic fields produced
 Krishna taught in the Bhagavadgita that by the coils surrounding the plasma as well
even women, vaishyas, and shudras who as by a current driven in the plasma itself,
were born of sin could seek refuge in him. help in confining the hot plasma away from

23
PTS2024/AIOMT/102023/07
surrounding material walls. The plasma is Supplementary notes:
heated to fusion relevant temperatures by
injecting high energy neutral particle beam PM SHRI Schools (PM Schools for Rising
or radio frequency waves from outside which India)
get absorbed in the plasma  The scheme will provide leadership in their
respective regions in providing high-quality
education in an equitable, inclusive and
50. Correct Option: (a) joyful school environment that takes care of
Explanation: the diverse background, multilingual needs,
and different academic abilities of children
 Option (a) is correct.
and makes them active participants in their
Supplementary notes: own learning process as per the vision of
NEP 2020.
Methods and application of Plant
biotechnology  Under the scheme there is provision of
setting up of More than 14500 PM SHRI
 Bacillus thuringiensis (Bt) is a gram- Schools (PM Schools for Rising India) by
positive spore-forming soil bacterium that
strengthening the existing schools from
synthesizes crystalline (Cry) protein, which
amongst schools managed by Central
is toxic and causing pathogenicity against
government/State/UT Government/local
mainly three insect orders: Coleoptera,
Diptera, and Lepidoptera and also against bodies.

RE
other invertebrates such as nematodes.  The duration of scheme is proposed to be
These crystalline protein inclusions, i.e., from 2022-23 to 2026-27; after which it
G-endotoxins are successfully used as a bio- shall be the responsibility of the States/UTs
control agent against insect pests. to continue to maintain the benchmarks
achieved by these schools. More than 20
lakh students are expected to be direct
51. Correct Option: (d)
beneficiaries of the scheme. The total cost
O
Explanation: of the project will be Rs. 27360 crore spread
over a period of 5 years which includes
 Both statements are correct.
central share of Rs. 18128 crores.
Supplementary notes:
SC
Pravasi Kaushal Vikas Yojana (PKVY) 53. Correct Option: (a)
 PKVY is a skill development initiative by
Explanation:
the Ministry of External Affairs. The MEA
and the Ministry of Skill Development and  Option (a) is correct: The platform aims to
Entrepreneurship (MSDE) had signed a pool the knowledge and synergise the efforts
Memorandum of Understanding (MoU) for of various organisations for the promotion
its implementation. and adoption of clean, energy technologies
 The scheme will be implemented by and practices.
GS

National Skill Development Corporation  It has been acknowledged that the


(NSDC) through its training partners in development of the SME sector requires an
consultation with MEA and MSDE. integrated approach that simultaneously
 PKVY is a skill development initiative of the addresses the technological, capacity,
MEA in partnership with the MSDE and is and financial needs of MSMEs.
aimed at training and certification of Indian  To improve the energy efficiency of the
workforce keen on overseas employment MSME sector, BEE, has undertaken
in select sectors and job roles, in line
several initiatives to maximise
with international standards, to facilitate
effectiveness and widespread adoption
overseas employment opportunities.
of energy efficient technologies.
Supplementary notes:
52. Correct Option: (b)
SAMEEEKSHA (Small and Medium
Explanation:
Enterprises Energy Efficiency Knowledge
 Option (b) is correct: The Cabinet has been Sharing)
approved a newly central sponsored scheme
called PM SHRI on 7 September, 2022. These  The Ministry of Power is celebrating Energy
schools will showcase the implementation Conservation Week under Azadi Ka Amrit
of the National Education Policy 2020 and Mahotsav from 8th to 14th December 2021.
emerge as exemplar schools over a period As part of celebrations, Bureau of Energy
of time, and also offer leadership to other Efficiency (BEE) under the Ministry of
schools in the neighborhood. Power has organized various programs.

PTS2024/AIOMT/102023/07 24
 The Bureau of Energy Efficiency (BEE) 54. Correct Option: (a)
organized an Interactive Workshop on
Outcomes of Energy and Resource Mapping Explanation:
of Ministry of Micro, Small & Medium  Statement 2 is incorrect: The tribal
Enterprises (MSME) Clusters at India Infant Mortality Rate (IMR) during the
Habitat Centre on December 11, 2021, period of 1988-2014 has halved, reduced
during “National Energy Conservation from 90 to 44.
Week: 8th–14th December 2021”, marked
Supplementary notes:
as Iconic Week under “Azadi ka Amrit
Mahotsav”. A presentation on findings of National Tribal health mission
‘Energy and Resource Mapping of MSME  For the first time since independence, a
clusters in India’ was given by BEE official, tribal President has become a reality in
regarding 8 sectors and 40 clusters. India. This is a very positive signal given
 The MSME sector is the backbone of large to the tribal people by the Narendra Modi
industrial units and generates employment government. On this International Day
opportunities in rural as well as urban areas. of the World’s Indigenous Peoples, let us
The MSME sector is consistently widening explore how this symbolic gesture can be
its domain across sectors of the economy as turned into a health revolution for the tribal
people of India.
diverse range of products and services are

RE
now being produced to meet the demands of  Few people are aware that nearly 11 crore
domestic as well as global markets. tribal people (enumerated as Scheduled
Tribes (ST) in the Census of India (2011)
 The Bureau of Energy Efficiency (BEE)
live in India. They constitute 8.6% of India’s
and the Ministry of MSME have together
population, the second largest number of
taken a number of initiatives to ensure the tribal people in any country in the world. The
growth of this sector in an energy-efficient Prime Minister and the national leadership
O
and environmentally-friendly way. To
ensure synergy among various players in
of the Bharatiya Janata Party are aware of
this fact, which is one of the reasons why
the MSME sector, BEE and the Ministry of they have chosen a tribal woman for the
MSME have also promoted a collaborative highest position in the country.
platform--”SAMEEEKSHA” (Small and
SC
 A study published in The Lancet, titled
Medium Enterprises Energy Efficiency
‘Indigenous and Tribal Peoples’ Health’
Knowledge Sharing).
(2016), found that India held the inglorious
 BEE appreciated the efforts of the expert distinction of having the second highest
agencies who conducted sectoral studies infant mortality rate for the tribal people,
for Bureau to facilitate policy development next only to Pakistan. This is not an
for MSMEs. India has aimed to reduce honourable position.
45% of its emission intensity, raising our
Findings
GS

bars to fight climate change. It is essential


to plan the roadmap with greater vision  Firstly, tribal people are concentrated in 809
inclusive of adoption to energy transition blocks in India. Such areas are designated
and advancement to latest technologies in as the Scheduled Areas. However, the more
unexpected finding was that half of India’s
all economic sectors.
tribal population, nearly five and a half
 The workshop was aimed at discussing crore, live outside the Scheduled Areas, as a
the findings of the work being undertaken scattered and marginalised minority. They
by BEE in 9 select sub-sectors in terms of are the most powerless.
energy and resource mapping.
 Second, the health status of tribal people
 Representatives from various sectors like has certainly improved during the last 25
Foundry and Forging, Steel Re-Rolling and years as seen in the decline in the under-
Paper, Glass, Refractory and Chemicals, five child mortality rate from 135 in 1988
Pharma and Bricks, Institute of Indian in the National Family Health Survey
Foundrymen, Association of Indian Forging (NFHS)-1 to 57 in 2014 (NFHS-4). However,
Industry, Steel Manufacturing Association, the percentage of excess of under-five
Paper Mills Association, Karnal Chemical morality among STs compared to others has
Cluster, Glass Syndicate Firozabad, Bulk widened.
Drug Manufacturers Association, All India  Third, child malnutrition is 50% higher in
Brick & Tile Manufacturers Federation tribal children: 42% compared to 28% in
were present during the event. others.

25
PTS2024/AIOMT/102023/07
 Fourth, malaria and tuberculosis are three government health expenditure on tribal
to 11 times more common among the tribal people becomes equal to the stated goal
people. Though the tribal people constitute of the National Health Policy (2017), i.e.
only 8.6% of the national population, half 2.5% of the per capita GDP.
of the total malaria deaths in India occur
among them.
55. Correct Option: (b)
 Fifth, while malnutrition, malaria and
mortality continue to plague tribal people, Explanation:
gradually, the more difficult to treat non-  Statement 1 is incorrect: An overseas
communicable diseases such as hypertension elector is a person who is a citizen of India
and diabetes, and worse, mental health
and who has not acquired citizenship of any
problems such as depression and addiction
other country and is otherwise eligible to be
leading to cancer and suicide are increasing.
registered as a voter.
These threaten the health and survival of
tribal adults. Supplementary notes:
 Sixth, tribal people heavily depend Voting Rights to Overseas Electors
on government-run public health care
institutions, such as primary health centres  An overseas elector is a person who is a
and hospitals, but there is a 27% to 40% citizen of India and who has not acquired
deficit in the number of such facilities, citizenship of any other country and is

RE
and 33% to 84% deficit in medical doctors otherwise eligible to be registered as a
in tribal areas. Government health care voter.
for the tribal people is starved of funds as  The Citizenship Act of 1955 prescribes five
well as of human resource. We found them ways of acquiring citizenship, viz, birth,
demoralised and inefficient. descent, registration, naturalisation and
 Seventh, there is hardly any participation incorporation of territory.
of the tribal people – locally or at the State
O
or national level – in designing, planning or
delivering health care to them.
56. Correct Option: (a)
 The official policy of allocating and spending Explanation:
an additional financial outlay, called Tribal  Statement 1 is correct: Variation in
SC
Sub-Plan (TSP), equal to the percentage density and gravity cause the movement of
of the ST population in the State, has surface and deep-water currents.
been completely flouted by all States. As
estimated for 2015-16, annually Rs. 15,000  Statement 2 is incorrect: Cold currents
crore should be additionally spent on tribal are usually found on the west coast of the
health. No accounts or accountability exist continents in the low and middle latitude.
on this. No one knows how much was spent Supplementary notes:
or not spent.
Water currents
GS

 These issues have persisted as there is no


separate data on tribal people’s health or  Surface currents constitute about 10 per
healthcare or on the money spent. cent of all the water in the ocean, these
 The committee was also asked by the waters are the upper 400 m of the ocean.
Government of India to prepare a road  Deep water currents make up the other 90 per
map for the future, which it did. This road cent of the ocean water. These waters move
map includes many recommendations, around the ocean basins due to variations in
but the three most important broad the density and gravity. Deep waters sink
recommendations are the following. into the deep ocean basins at high latitudes,
 Firstly, launch a National Tribal Health where the temperatures are cold enough to
Action Plan with a goal to bring the cause the density to increase.
status of health and healthcare at par  Ocean currents can also be classified based
with the respective State averages in the on temperature: as cold currents and warm
next 10 years. currents:
 Second, the committee suggested nearly  cold currents bring cold water into warm
80 measures to address the 10 priority water areas. These currents are usually
health problems, the health care gap, the found on the west coast of the continents
human resource gap and the governance in the low and middle latitudes (true in
problems. both hemispheres) and on the east coast
 Third, the committee suggested allocation in the higher latitudes in the Northern
of additional money so that the per capita Hemisphere;

PTS2024/AIOMT/102023/07 26
 warm currents bring warm water hemisphere, where the continents within
into cold water areas and are usually the high latitudes have a broad east west
observed on the east coast of continents spread.
in the low and middle latitudes (true  On its poleward side, it merges into Arctic
in both hemispheres). In the northern
tundra of Canada & Eurasia at around
hemisphere they are found on the west
Arctic Circle.
coasts of continents in high latitudes.
 Southwards, the climate becomes less severe
& fades into the temperate Steppe climate.
57. Correct Option: (c)
 Predominant vegetation of this Siberian
Explanation: or sub-arctic type of climate is evergreen
 Statement 1 is correct: The Siberian type coniferous forests that stretch in a great,
of climate is only experienced in northern continuous belt across North America,
hemisphere. Europe & Asia.

 Statement 2 is correct: British type of  The greatest single band of the coniferous
climate region is influenced with a lot of forest is Taiga (a Russian word for coniferous
cyclonic activity. forest) in Siberia.
 Statement 3 is correct: Central California,  In Europe, this climate & forests are mainly
south-eastern and southwestern Australia confined to Northern Europe, Sweden &

RE
have Mediterranean climate. Finland.

Types of climates
58. Correct Option: (b)
British Type of Climate
 The Westerlies have a constant influence
Explanation:
on the cool temperate western margins  Statement 1 is incorrect: The Himalayan
throughout the year. Mountain range was formed due to the

O
They are also regions with a lot of cyclonic collision of Eurasian plate and the Indo-
Australian plate.
activity, which is typical of Britain, so they
are said to have a British type of climate.  Statement 2 is correct: North plain
 Rainfall is common throughout the year. formation took place because of Himalayan
SC
upliftment and subsidence of the northern
 They experience cool, rainy winters and flank of the peninsular plateau.
warm, rainy summers.
 The weather is also extremely Supplementary notes:
unpredictable. Formation of Himalaya
 This climate is also called Cool Temperate  The Gondwana land included India,
Western Margin Climate or North-West Australia, South Africa, South America and
European Maritime Climate. Antarctica as one single land mass.
GS

Mediterranean climate  The convectional currents split the crust


 Mediterranean climate occurs around into a number of pieces, thus leading to the
Mediterranean Sea, along the west coast of drifting of the Indo-Australian plate after
continents in subtropical latitudes between being separated from the Gondwana land,
30° - 40° latitudes e.g. — Central California, towards north.
Central Chile, along the coast in south  The northward drift resulted in the
eastern and south western Australia. collision of the plate with the much larger
 These areas come under the influence of Eurasian Plate. Due to this collision, the
sub-tropical high in summer and westerly sedimentary rocks which were accumulated
wind in winter. in the geosyncline known as the Tethys
 Hence, the climate is characterised by hot, were folded to form the mountain system of
dry summer and mild, rainy winter. western Asia and Himalaya.

 Monthly average temperature in summer is  The Himalayan mountain range was formed
around 25° C and in winter below 10°C. due to the collision of two tectonic plates,
namely the Eurasian plate and the Indo-
 The annual precipitation ranges between Australian plate, just a few million years
35 - 90 cm.
ago.
Siberian type climate  When a mountain range is formed due to
 The Cool temperate continental (Siberian) the collision of two tectonic plates, they are
climate is only experienced in northern called fold mountains.

27
PTS2024/AIOMT/102023/07
 The Himalayan uplift out of the Tethys Sea  Schemes like employment guarantee
and subsidence of the northern flank of the (MGNREGA) that boost economic activity
peninsular plateau resulted in the formation but increase government expenditure,
of a large basin. ultimately forming a part of the fiscal
deficit, are a part of Above-the-Line fiscal
 In due course of time this depression, measures.
gradually got filled with deposition of
 Tax Measures like tax reliefs and cuts
sediments by the rivers flowing from the increase government debts by reducing
mountains in the north and the peninsular revenues are also a part of Above-the-Line
plateau in the south. A flat land of extensive fiscal measures.
alluvial deposits led to the formation of the
 Direct/ Targeted Transfers from government
northern plains of India. to citizens like DBT also form a part of
 And since the Himalayan Mountain range government liabilities directly and are a
was formed just a few million years ago, part of Above-the-Line fiscal measures.
they are known as young mountains.  Asset purchase may have little or no upfront
impact on the fiscal deficit although they
can later increase debt or reduce liquidity.
59. Correct Option: (c)
Explanation: 60. Correct Option: (a)
Option (c) is correct: Loans and Credit

RE
 Explanation:
Guarantees are not reflected in government
debt immediately.  Statement 2 is incorrect: Chhattisgarh
is the poorest, with nearly 40 % of the
Supplementary notes: population living below the national poverty
line.
‘Above-the-Line’ fiscal measures
Supplementary Notes:
 “Above the line” measures are revenue and
O
expenditure measures, which have fiscal Poverty in India
implications.  In India percentage of poor in scheduled
 “Above the line” measures refer to increases castes is less than that of scheduled tribes.
in government expenditures and reductions  The multidimensional poverty index (MPI)
SC
in tax revenues. 2018, found that 50 per cent of all tribal in
the country are poor as are 33 per cent of
 Above the- line measures include those for Scheduled castes or Dalits and 33 per cent
which full cost is reflected in the fiscal deficit, of Muslims.
government debt, and increased borrowing
 Chhattisgarh is the poorest, with nearly 40
needs in the short term. These measures
% of the population living below the national
include additional spending, capital grants poverty line, followed by Jharkhand (37
and targeted transfers or tax measures. per cent) on the other hand; Bihar is the
Below-the-line measures are those that 5th poorest state in India. Half of Bihar’s
GS


involve the creation of assets, like equity population is living below the poverty line.
injections, loans and asset purchase. Below-  The poverty level in this state is around
the-line may have little or no upfront impact 33.74%.
on the fiscal deficit although they can later  The reasons behind poverty are both
increase debt or reduce liquidity. institutional and technical, illiteracy and
 Increasing government expenditures (such structural and institutional factors of
agriculture.
as subsidized food programmes or health
facilities etc. costs are reflected in the fiscal
deficit, government debt, and increased 61. Correct Option: (a)
borrowing needs in the short term and
hence they are Above the line measures. Explanation:
 Option (a) is correct.
 Capital Grants are a part of Capital
Expenditure of a government in the Supplementary notes:
Government’s budget, and are used in the
Global Financial Crisis
calculation of the fiscal deficit.
 The Short-term debts have to be returned at
 Loans and Credit Guarantees are not a shorter interval. This renders an economy
reflected in government debt immediately. vulnerable if the economy is already facing
However, they may increase government economic crisis as it has an obligation to
liability in the long run. It is thus a part of return the debt as well as interest payments
Below-the-Line fiscal measures. (debt service).

PTS2024/AIOMT/102023/07 28
 Example: Many economies like Mexico, Montreal Protocol
Argentina etc who faced crisis were seen  The Montreal Protocol on Substances that
to have borrowed large amounts of short- Deplete the Ozone Layer is the landmark
maturity debt. So, the ideal scenarios are multilateral environmental agreement that
not to depend upon short-term debt during regulates the production and consumption
crisis. of nearly 100 man-made chemicals referred
to as ozone depleting substances (ODS).
 Opening up to the foreign banks and
depending upon them is not a good idea  The treaty is universal with 197 member
during crisis. It has been empirically countries.
observed in World Bank Research that in  It is legally binding with penalties for
many cases (like during Global Financial countries that flout its provisions.
Crisis 2007-08) that foreign banks reduced  And it is fully funded, meaning that poorer
their lending during crisis. Many of them countries that might not have been able to
choose to retreat from cross-border banking meet its targets to phase down chemicals
in general, including through cutting back received assistance from richer ones.
on new entry. Kyoto Protocol
 Capital account convertibility would mean  The 1997 Kyoto Protocol – an agreement
that there is no restriction on conversion under the United Nations Framework
of the domestic currency into a foreign Convention on Climate Change (UNFCCC)

RE
currency. It is recognised that capital flows – is the world’s only legally binding treaty
are sensitive to macroeconomic conditions. to reduce greenhouse emissions.

 Any deterioration in fiscal conditions,


inflation management, balance of payments, 63. Correct Option: (a)
or any other economic crisis may cause a
Explanation:
cessation or reversal of capital flows. This
Option (a) is incorrect: Green Climate
O
might make the economy vulnerable if it
already suffering from economic crisis.

Fund (GCF) was established to reduce
greenhouse gas (GHG) emissions in
developing countries only.
62. Correct Option: (d) Supplementary notes:
SC
Explanation: Important International Environmental
 Option (d) is correct. Organizations

Supplementary notes: Green Climate Fund


 GCF is a unique global platform to respond to
International Environmental Treaties climate change by investing in low-emission
and climate-resilient development.
Kigali Amendment
GS

 GCF was established by 194 governments


 The Kigali Amendment to the Montreal to limit or reduce greenhouse gas (GHG)
Protocol is an international agreement emissions in developing countries, and
to gradually reduce the consumption and to help vulnerable societies adapt to the
production of hydrofluorocarbons (HFCs). unavoidable impacts of climate change.
 It is a legally binding agreement designed to  Given the urgency and seriousness of this
create rights and obligations in international challenge, GCF is mandated to make an
law. ambitious contribution to the united global
response to climate change.
Paris Agreement
Global Environment Facility
 The Paris Agreement often referred to as the
 The Global Environment Facility (GEF),
Paris Accords or the Paris Climate Accords,
established on the eve of the 1992 Rio Earth
is an international treaty on climate change. Summit, is a catalyst for action on the
Adopted in 2015, the agreement covers environment — and much more.
climate change mitigation, adaptation, and
 The GEF is a unique partnership of 18
finance.
agencies — including United Nations
 The Agreement is a legally binding agencies, multilateral development banks,
international treaty. It entered into force national entities, and international NGOs
on 4 November 2016. Today, 194 Parties — working with 183 countries to address
(193 States plus the European Union) have the world’s most challenging environmental
issues.
joined the Paris Agreement.

29
PTS2024/AIOMT/102023/07
 It is a financial mechanism for five major  Black carbon absorbs solar energy, it warms
international environmental conventions: the atmosphere. When it falls to earth with
the Minamata Convention on Mercury, precipitation, it darkens the surface of snow
the Stockholm Convention on Persistent and ice, reducing their albedo (the reflecting
Organic Pollutants (POPs), the United power of a surface), warming the snow, and
Nations Convention on Biological Diversity hastening melting.
(UNCBD), the United Nations Convention  It gets emitted from gas and diesel engines,
to Combat Desertification (UNCCD) and coal-fired power plants, and other sources
the United Nations Framework Convention that burn fossil fuel. It comprises a
on Climate Change (UNFCCC). significant portion of particulate matter or
PM, which is an air pollutant.
European Environment Agency
 The European Environment Agency (EEA)
is the agency of the European Union (EU) 65. Correct Option: (a)
which provides independent information on
Explanation:
the environment.
 Pair 1, 3 & 4 are incorrectly matched.
 The member states of the European
Union are members; however other states 1. Metals - Associated Disease
may become members of it by means of
2. Mercury - Minamata disease
agreements concluded between them and

RE
the EU. 3. Cadmium - Itai Itai

Intergovernmental Panel on Climate 4. Lead - Anaemia


Change (IPCC) 5. Arsenic - Black foot disease
 The Intergovernmental Panel on Climate
Change (IPCC) is an intergovernmental Supplementary notes:
body of the United Nations charged with Metal poisoning and Diseases
O
advancing scientific knowledge about
 Minamata disease (M. d.) is methylmercury
anthropogenic climate change.
(MeHg) poisoning that occurred in humans
 It was established in 1988 by the World who ingested fish and shellfish contaminated
Meteorological Organization (WMO) and the by MeHg discharged in waste water from a
SC
United Nations Environment Programme chemical plant.
(UNEP) and endorsed by the UN later that  Itai-itai disease was the name given to
year. the mass cadmium poisoning of Toyama
Prefecture, Japan, starting around 1912.
64. Correct Option: (d) The term “itai-itai disease” was coined by
locals for the severe pains people with the
Explanation: condition felt in the spine and joints.
 Option (d) is correct.
GS

Supplementary notes: 66. Correct Option: (c)


Black Carbon Explanation:
 Black Carbon (BC) is a short-lived pollutant  Statement 3 is incorrect: Opposing the
that is the second-largest contributor to Non-Cooperation Movement, Tagore told
Gandhi that ‘the whole world is suffering
warming the planet behind carbon dioxide
today from the cult of a selfish and short-
(CO2).
sighted nationalism. India has all down her
 Unlike other greenhouse gas emissions, BC history offered hospitality to the invader
is quickly washed out and can be eliminated of whatever nation, creed or colour. I have
from the atmosphere if emissions stop. come to believe that, as Indians, we not only
 Unlike historical carbon emissions it is have much to learn from the West but that
also a localised source with greater local we also have something to contribute. We
impact. dare not therefore shut the West out.

 Black carbon is a kind of an aerosol. Among Supplementary notes:


aerosols (such as brown carbon, sulphates),
Social-political aspects of freedom
Black Carbon has been recognized as the
second most important anthropogenic agent  Gandhi and Tagore love their national
for climate change and the primary marker culture and civilization but they differ to
to understand the adverse effects caused by the extent. Gandhi devised his scheme of
air pollution. education strictly for Indian children. He

PTS2024/AIOMT/102023/07 30
planned education according to the needs Supplementary notes:
of Indian society and Indian culture. But
Tagore had a much broad vision. Though Bharatendu Harishchandra
he was also a true lover of Indian art and  Bharatendu was the man behind what is
culture, he wanted to bring about harmony known as the ‘Hindi vernacular nationalism’.
between the east and the west. His scheme He was the driving force behind the
of education therefore included western art mass movement that culminated in the
and literature as well. Thus, he endeavored popular slogan “Hindi, Hindu, Hindustan”.
for the fusion and harmonious synthesis  He founded and edited one of India’s first
of all the cultures. He was out and out women’s journals, Bala Vodhini. Bharatendu
cosmopolitan in views. His Shantiniketan is wrote under the pen name, ‘Rasa’, probably
a cosmopolitan educational institution. borrowing from Natya Shastra.
 Both of them thought colonialism was an  He was also the editor of two popular journals
internal psychological construct and paid at the time, Kavivachansudha (founded
less attention to the externally superimposed 1868) and Harishchandrachandrika
domination of the alien power which, (founded 1873), which were known to voice
they felt, would crumble once the people the literary sentiment of the age.
were aroused. Both of them prescribed a
philosophy of life based on Indian values, 68. Correct Option: (d)
ethics and customs.

RE
Explanation:
 Both the thinkers invoked ancient
 Option (d) is correct.
Indian wisdom as a weapon to fight not
only British colonialism, but also the Supplementary notes:
ignorance and superstitions prevalent
Begam Rokey Sakhawat Hossain
in Indian society. For them, political
emancipation was not the disappearance  From the early twentieth century, Muslim
of the external fetters only. India needed women played a notable role in promoting
O
to be freed from the shackles that bound education among women. Rokeya Sakhawat
Hossain was one such Muslim female social
the mind. Only education, they thought,
reformer who acquired education with the
could deliver the nation spiritually. dream of bringing about a change in the
Both Gandhi and Tagore were precise in condition of women.
SC

identifying the central fault with modernity  She learnt Bangla and English and later
— the fact that it unnecessarily complicated became a writer. She wrote a story ‘Sultana’s
even simple things because of its in-built Dream’ in 1905.
drive towards power and the multiplication  In her book she talks of an imaginary world,
of wants. Importantly, they both saw lady land, where women are free and are
‘modernity’ as being not only the source of treated equally.
a myriad crises for humanity but being in
 Begum Rokeya Sakhawat Hossain started
terminal crisis itself.
GS

schools for Muslim girls in Patna and


Calcutta.
67. Correct Option: (b)  She was a fearless critic of conservative
ideas, arguing that religious leaders of every
Explanation: faith accorded an inferior place to women.
 Option (b) is correct: Bharatendu was  In 1929, she wrote a story book named
barely seven years old when the revolt ‘Avarodhbasini’ in which she attacked on the
of 1857 took place. Called the first war of extreme forms of purdah that endangered
Independence, it had left quite an impression women’s lives and thoughts.
on young Bharatendu
 He wrote: “Andher Nagari, Chaupat 69. Correct Option: (a)
Raja, taka ser bhaaji, take ser khaja”, (A
Explanation:
dark city, a failing king, a penny for
sweets and a penny for onion rings).  Option (a) is correct.

 These lines from Andher Nagari (city Supplementary notes:


of darkness), one of his most famous Pandita Ramabai
plays, effectively portray a city that falls
 Ramabai was an Indian reformer who was
into darkness due to the lack of sound
critical of the conception of male dominance
administration, is known to be a popular
which led to the suppression and subjugation
allegory for British rule in India. of women.

31
PTS2024/AIOMT/102023/07
 Her book The High-Caste Hindu Woman was Pradesh, at a height of 1800–3000
based on the miseries of girls and women in metres, receiving a minimum rainfall
high-caste Hindu families. of 200 cm.
 Ramabai understood that Hindu patriarchy  In the South, it is found in parts of
has placed women within the domestic the Nilgiri Hills, the higher reaches
sphere as a wife, mother, or housewife of Kerala.
according to their role in homemaking.
 Himalayan Moist Forests are found in
 She held the view that patriarchal ideology Jammu & Kashmir, Himachal Pradesh,
in India regarded widowhood a punishment Uttarakhand, and Northern hilly parts of
for horrible crimes committed by a woman Bengal.
in her previous birth, including disobedience
 Swamp Forests are found along the
or disloyalty to her husband. In Poona,
Andaman and Nicobar Islands and the delta
Ramabai, working towards the aim of
area of the Ganga and the Brahmaputra.
gender reform, formed the Arya Mahila
Samaj in 1882 and established branches
throughout the Mumbai region. The object 71. Correct Option: (c)
of this institution was to promote education
among native women and to discourage Explanation:
child marriage.  Statement 1 is incorrect: Madame H.P.
Ramabai’s most popular academic venture, Blavatsky and Colonel Olcott founded the

RE

The High-Caste Hindu Woman, contained a Theosophical Society in New York in 1875
critical account of miseries that were faced who were inspired by Indian thought and
by girls and women in the domain of a high- culture. Theyarrived in India in January
caste Hindu joint family system. 1879, and establishedheadquarters at
Adyar near Madras.
 Pandita Ramabai considered women of
lower castes to be much better off in terms  Statement 2 is incorrect: Gangabai
of self-reliance and freedom since they were (Maharani Tapaswini) set up Mahakali
O
obliged to depend upon themselves. Pathshala in Calcutta in 1893 to propagate
female education without foreign influence.
 It is not wrong to say that Ramabai entered
into the feminist discourse through her book  Statement3 is incorrect: Deoband
Stri Dharma Niti (Morals for women). This Movement was established in Deoband in
SC
book turned out to be a guide of women’s Saharanpur district (United Provinces) in
morality, asking illiterate, ignorant women 1866 by Mohammad Qasim Nanotavi and
to recast themselves in a more cultural Rashid Ahmed Gangohi to train religious
mould through self-reliance and self- leadersfor the Muslim community.
education. Supplementary notes:
Theosophical Society
70. Correct Option: (d)
 The Theosophical Society was founded by
GS

Explanation: Madame Blavatsky and Col. Olcott in 1875


 All statements are correct. in New York.
 It was only in 1879, that this ideology gained
Supplementary notes:
its roots in the Indian culture and Society.
Types of Forests  It was crystallised in the Madras Presidency
 Moist Evergreen Forests are found in with its headquarters in Adayar. The
southern India along the Western Ghats, movement was popularised by Annie Besant
Andaman and Nicobar Islands and north- in India.
eastern region.  Theosophy was based on three principles:
 Moist Decidous Forests: These forests are  Universal brotherhood.
found in the north-eastern states along
the foothills of Himalayas, eastern slopes  The study of comparative religion and
of the Western Ghats and Odisha. philosophy.

 Thorn Forests: The forests occur in the  Investigations into the natural
areas that receive annual rainfall less laws which seek to understand the
than 50cm. unexplained mystic laws.

 Montane Wet Temperate Forests: Occur Mahakali Pathshala


in the northern and southern India.  The Mahakali Pathshala in Calcutta was
 In the North, it is found in the region founded by Maharani Tapaswini in the year
to the east of Nepal into Arunachal 1893.

PTS2024/AIOMT/102023/07 32
 She was one of the first persons to 73. Correct Option: (d)
concentrate on girls, women education.
Explanation:
 She has fought along with Rani Laxmibhai
in the revolt of 1857.  Both statements are correct

Deoband Movement Supplementary notes:


 The Deoband Movement was organised by Non-cooperation Movement & Mahatma
the orthodox section among the Muslim Gandhi
ulema as a revivalist movement with
the twin objectives of propagating pure  Gandhi’s attitude changed once the harsh
teachings of the Quran and Hadis among terms imposed by the Allied Powers on
Muslims and keeping alive the spirit of Turkey in the Treaty of Serves (May 1920)
jihad against the foreign rulers. became known.

 The Deoband Movement was established  It was hardened by the publication of Hunter
in Deoband in Saharanpur district (United Commission Report on the disturbances
Provinces) in 1866 by Mohammad Qasim in Punjab connected with the Rowlatt
Nanotavi (1832-80) and Rashid Ahmed Satyagraha.
Gangohi (1828- 1905) to train religious  This Report protected the officers of
leaders for the Muslim community. the British government by means of an
Indemnity Act, acquitted O’Dwyer of blame,
and the British House of Lords rejected the

RE
72. Correct Option: (b) censure passed on Dyer..
Explanation:  The non-cooperation programme was
 Statement 1 is incorrect: The Haileybury to be implemented in stages with civil-
College in Oxford, a rival of Fort William, to disobedience and non-payment of taxes
train the Company’s servants to be worthy kept for the second stage, only if swaraj did
bureaucrats. not come within a year and the government
resorted to repression.
Supplementary notes:
O
Education during British India 74. Correct Option: (c)
 College of Fort William, the ‘Oxford of the
Explanation:
East’ evolved into an institution not only
SC
for training civil servants, but also for  Both statements are correct.
patronizing literary and linguistic research
and Orientalist scholarship in general.
Supplementary notes:

 The Haileybury College in Oxford, a rival of India’s Freedom Struggle (2nd Phase)
Fort William, aims to train the Company’s  In the Madras Congress session in
servants to be worthy bureaucrats. December 1927 where the resolution to
 This College was to ensure that the ‘boys’, boycott the Simon Commission was taken,
the future servants of the Company, were Jawaharlal Nehru, supported by Subhas
GS

indoctrinated ‘well’ by the Cambridge Bose, moved a resolution to make purna


clergymen before they were sent off to India swaraj or complete independence the goal
and placed under the influence of Orientalist of the Congress struggle.
scholars at Fort William.  And yet, the Congress decided to embark
The Thuggee Act XXX of 1836 on the task of drafting a constitution for
‘dominion status’.
 The passing of The Thuggee Act XXX of 1836
and the relentless pursuit not only of thugs  This was, in part, a response to the challenge
but of all groups of religious mendicants thrown to the leaders of the Swaraj Party
by Sleeman, the superintendent of thugee by Secretary of State Lord Birkenhead to
operations, eventually ‘purged India of this ‘produce a constitution that carried behind
great pollution’ (Kaye 1853: 376). it a fair measure of general agreement
among the great peoples of India’ and in part
 Sleeman, who had ‘discovered’ the notorious Gandhi’s disagreement with Jawaharlal
thugs, wrote a long self-congratulatory Nehru’s snap independence decision passed
account of his successful operation (Sleeman during his absence.
1839 in Harlow and Carter 1999: 81–87).
 No special representation in the Central
 The provision of the 1836 Act of legally trying Parliament was allowed except for the
‘criminal communities’ and the knowledge Muslims in Bengal and the non-Muslims
compiled by the criminal department would, in the North-West Frontier Province. The
after the Revolt of 1857, provide the basis provincial councils, on the other hand,
for the demarcation of ‘criminal castes’ and would have reserved seats for minority
‘criminal tribes communities.

33
PTS2024/AIOMT/102023/07
 The independence pledge that the Indian  And authorities who were then brought
National Congress presented to the world together in a circle of friendship and
on 26 January 1930 stated that the British homage. Instead of bolstering a bureaucratic
government in India had taken away the hierarchy, they helped to develop political
freedom of Indians and systematically hierarchies.
exploited them. It had ‘ruined India  Samudragupta did not wish to vanquish his
economically, politically, culturally and enemies merely for his own gratification.
spiritually’. After defeating them he permitted most
 Further, the pledge affirmed that as Indians of them to keep their domains within the
they considered it a sin ‘before man and God empire.
to submit any longer to a rule that has caused  They were to be protected by him, while he
this four-fold disaster to the country’. would share in their wealth. What he was
developing was a form of contract between
the tributary kings and himself as the
75. Correct Option: (b) overlord.
Explanation:
 Statement 2 is incorrect: Kushanas used 76. Correct Option: (a)
Roman gold and silver coins as high value
currency. Explanation:
Statement 1 is incorrect: Situational

RE

Supplementary notes: Poverty is a temporary type of poverty
Coinage System in Ancient India based on occurrence of an adverse event like
environmental disaster, job loss and severe
 In his empire many religious traditions health problem.
flourished: Persian Zoroastrian, Greek
and Roman paganism, Buddhism, Chinese  Statement 2 is incorrect: Relative Poverty
is defined from the social perspective,
Confucianism, Jainism and, above all, a
that is living standard compared to the
whole range of Vedic traditions. His coins
O
economic standards of population living
bear their proof, because they carry images
in surroundings. Hence it is a measure of
of deities from the various religions practised
income inequality.
in his kingdom.
 Statement 3 is incorrect: Generational
A large hoard of Roman coins found at
SC
 Poverty is handed over to individual and
Arikamedu and other places in south India, families from one generation to the one.
along with the archaeological remains of a
Roman colony, testify to the vigour of the Supplementary notes:
Indo-Roman trade.
Types of Poverty
 Owing to the absence of local gold coins in
 When a person’s/family’s income and
south India, Roman gold and silver coins
support is decreased due to a specific change
were also used as high-value currency,
– job loss, divorce, death, etc. While there
which would have encouraged hoarding.
GS

can be a domino effect caused by this one


 Only a quarter of the good-quality Roman significant change, families experiencing
coin hoards discovered in south India have Situational Poverty tend to remain hopeful,
been found in north India: the Kushan knowing that this is a temporary setback.
state issued its own gold coins, as valuable  Generational poverty is a term applied to
in metal terms as the Roman ones, and families who have experienced poverty for
simply melted down the Roman coins for at least two generations. It can affect every
their metal. aspect of a person’s life: physical, social,
Gupta Administration emotional and mental.

 For nearly a thousand years the Indian  Relative poverty is when households receive
monarchs, whether imperial or regional, 50% less than average household incomes.
So they do have some money but still not
had aimed at a concentration of all power
enough money to afford anything above
in their hands. Absolutist rulers were
the basics. This type of poverty is, on the
tempted to subdue provinces and regions to
other hand, changeable depending on the
their will through their bureaucracies, and
economic growth of the country.
the Mauryan were perhaps the greatest of
centralisers.  Absolute poverty is when household income
is below a certain level. This makes it
 The Gupta dominions were organised impossible for the person or family to meet
on a somewhat different model, the basic needs of life including food, shelter,
main difference being the principle of safe drinking water, education, healthcare,
decentralised administration. etc.

PTS2024/AIOMT/102023/07 34
77. Correct Option: (a)  Loess is in fact, fine loam, rich in lime, very
coherent and extremely porous.
Explanation:
 Water sinks in readily so that the surface is
 Statement 1 is incorrect: India’s currency always dry. Streams have cut deep valleys
notes printing presses at Nasik and Dewas through the thick mantle of soft loess and
are owned by the Government of India; two
badland topography may develop.
others Mysore and Salboni, are owned by the
RBI through its wholly owned subsidiary,  It is so soft that roads constructed through
Bharatiya Reserve Bank Note Mudran Ltd a loess region soon sink and their walls rise
(BRBNML). sharply.
Supplementary notes:  The most extensive deposit of loess is found
in north-west China in the loess plateau of
Indian currency design mechanism
the Hwang-Ho basin.
 Changes in the design and form of bank notes
and coins are decided by the Reserve Bank
of India (RBI) and the central government. 79. Correct Option: (c)
 Changes in the design of coins are the Explanation:
prerogative of the central government.
 Both statements are correct.
 Any change in design of a currency note has
to be approved by the RBI’s Central Board Supplementary notes:

RE
and the central government.
Isotherms
 The Governor draws his powers from
Section 7(3) of the RBI Act. He can exercise  The Isotherms are lines joining places
all powers and do all things that may be having equal temperature. In the northern
exercised and done by the RBI. hemisphere the land surface area is much
 Section 7(1) of RBI Act confers powers on larger than in the southern hemisphere.
the Centre to issue directions to the RBI  Hence, the effects of land mass and the
O
“from time to time” in the public interest
after consultations with the Governor.
ocean currents are well pronounced. In
January the isotherms deviate to the north
 The Coinage Act, 2011 gives the central over the ocean and to the south over the
government the power to design and mint continent. This can be seen on the North
coins in various denominations. In the case Atlantic Ocean.
SC
of coins, the role of the RBI is limited to the
distribution of coins that are supplied by  The presence of warm ocean currents, Gulf
the central government. Stream and North Atlantic drift, make the
Northern Atlantic Ocean warmer and the
 Indian currency notes are designed to reflect
the diverse cultural heritage of the country. isotherms bend towards the north.
They often feature images and motifs that  Over the land the temperature decreases
represent various aspects of Indian culture, sharply and the isotherms bend towards
including historical landmarks, prominent south in Europe.
GS

figures, and symbols of national pride.


 The effect of the ocean is well pronounced
 This cultural representation not only in the southern hemisphere. Here the
showcases India’s rich heritage but also
isotherms are more or less parallel to the
fosters a sense of national identity among
its citizens. For example, the new `500 and latitudes and the variation in temperature
`2000 notes issued in 2016 incorporated is more gradual than in the northern
images of the Red Fort and Mangalyaan, hemisphere. The isotherm of 20°C, 10°C,
India’s Mars Orbiter Mission, respectively, and 0°C runs parallel to 35°S, 45°S and
as a tribute to the country’s scientific and 60°S latitudes respectively.
historical achievements.

80. Correct Option: (b)


78. Correct Option: (c)
Explanation:
Explanation:
 Statement 1 is incorrect: Sustainable
 Both statements are correct. Access to Markets and Resources for
Supplementary notes: Innovative Delivery of Healthcare
(SAMRIDH) is supported by the United States
Loess Agency for International Development in
 The fine dust blown beyond the desert limits collaboration with Atal Innovation Mission
is deposited on neighbouring lands as loess. & Women Entrepreneurship Platform, NITI
It is a yellow, friable material and is usually Aayog, Principal Scientific Advisor to the
very fertile. Government of India.

35
PTS2024/AIOMT/102023/07
Supplementary notes: to the states and the centre synchronise.
This would mean that the voters will cast
SAMRIDH their vote for electing members of the LS
 SAMRIDH recognise that innovators and and the state assemblies on a single day, at
enterprises are a vibrant force with the the same time or in a phased manner as the
potential to change the Indian healthcare case may be.
landscape.
 SAMRIDH, with initiatives complimenting 82. Correct Optio: (a)
the public sector efforts, aims to support
private sector enterprises to effectively Explanation:
contribute towards India’s SDG aspirations,  Statement 2 is incorrect: CAG has no
including an integrated Health System control over issue of money and many
offering Universal Health Coverage (UHC).
departments are allowed to draw money by
 SAMRIDH has mobilized a capital pool of issuing cheque without specific authority
$300 million to offer both grant and debt from CAG.
financing provision to healthcare enterprises
and innovators, to augment their capacity Supplementary notes:
for production and supply of high impact Comptroller and Auditor-General of
health solutions.
India (CAG)
SAMRIDH won the P3 Impact Award 2022

RE

 Mandated by the Constitution of India,
led by the U.S. State Department’s Office
CAG promote accountability, transparency
of Global Partnerships, the University of
and good governance through high quality
Virginia Darden School of Business Institute
auditing and accounting and provide
for Business in Society, and Concordia.
independent and timely assurance to the
Legislature, the Public and the Executive,
81. Correct Option: (c) that public funds are being collected and
used effectively and efficiently.
O
Explanation:
 No minister can represent CAG in
 All statements are correct. parliament and no minister can be called
Supplementary notes: upon to take any responsibility for any
actions done by him.
SC
Simultaneous Elections  There shall be a Comptroller and Auditor-
 Law commission noted that simultaneous General of India who shall be appointed by
elections cannot be held within the the President by warrant under his hand
existing framework of the Constitution. and seal and shall only be removed from
Simultaneous elections may be conducted to office in like manner and on like grounds as
Lok Sabha and state Legislative Assemblies a Judge of the Supreme Court.
through appropriate amendments to the
 Every person appointed to be the
Constitution, the Representation of the
GS

Comptroller and Auditor-General of India


People Act 1951, and the Rules of Procedure
shall, before he enters upon his office, make
of Lok Sabha and state Assemblies.
and subscribe before the President or some
 Law commission noted that a no-confidence person appointed in that behalf by him, an
motion, if passed, may curtail the term of oath or affirmation according to the form set
Lok Sabha/ state assembly. It recommended out for the purpose in the Third Schedule.
replacing the ‘no-confidence motion’ with a
‘constructive vote of no-confidence’, through  The salary and other conditions of service
appropriate amendments. In a constructive of the Comptroller and Auditor-General
vote of no confidence, the government may shall be such as may be determined by
only be removed if there is confidence in an Parliament by law and, until they are so
alternate government. determined, shall be as specified in the
Second Schedule: Provided that neither the
 Simultaneous elections ensure that the salary of a Comptroller and Auditor-General
administrative machinery is engaged nor his rights in respect of leave of absence,
in development activities rather than pension or age of retirement shall be varied
electioneering.
to his disadvantage after his appointment.
 The “One Nation, One Election” idea
envisages a system where elections to all
state assemblies and the Lok Sabha will 83. Correct Option: (a)
have to be held simultaneously. This will Explanation:
involve the restructuring of the Indian
election cycle in a manner that elections  Option (a) is correct.

PTS2024/AIOMT/102023/07 36
Supplementary notes: Supplementary notes:
About XI schedule of constitution: Emergency provision:
 Schedule 11 of the Indian Constitution  Emergency provision is a unique feature of
contains provisions that specify the Indian Constitution that allows the Centre to
Panchayats’ powers, authority, and assume wide powers so as to handle special
responsibilities in order for them to function situations. In emergency, the Centre can
as local governments. take full legislative and executive control of
 The Eleventh Schedule was added by the 73rd any state. Emergency provision also allows
Amendment Act of 1992. It has 29 subjects the Centre to curtail or suspend freedom
such as Agriculture, Animal Husbandry, of the citizens. Existence of emergency
Fishery, etc. provision in the Constitution is a big reason
 11th Schedule contains the provisions why academicians are hesitant to call Indian
that specify the powers, authority and Constitution as fully federal.
responsibilities of Panchayats. This schedule  Fundamental Rights under Article 19
was added by the 73rd Amendment Act of are automatically suspended and this
1992. suspension continues till the end of the
 It has 29 matters including land emergency.
improvement, library and cultural activity.
 President may dissolve the State Legislative
All other subjects are part of XII schedule.

RE
Assembly or put it under suspension. He
may authorise the Parliament to make laws
84. Correct Option: (a) on behalf of the State Legislature.

Explanation:
 Option (a) is correct. 86. Correct Option: (d)
Supplementary notes: Explanation:

Supreme court of India:


O  All statements are correct.

 The Supreme Court of India is the supreme Supplementary notes:


judicial authority of India and the highest
Global Biofuels Alliance (GBA)
SC
court of the Republic of India under the
constitution.  The Global Biofuels Alliance (GBA)
announcement was made by Indian Prime
 It is the highest constitutional court and
has the final decision in all legal matters Minister on the sidelines of the G20 Summit
pertaining to common law which excludes held in New Delhi.
legal authority over personal laws. It also  It is an initiative to develop an alliance of
has the power of judicial review. governments, international organisations
 The Chief Justice of India is the Head of and industry to facilitate adoption of biofuels
GS

the Supreme Court, which consists of a by bringing together the biggest consumers
maximum of 34 judges, and has extensive and producers of biofuels and positioning
powers in the form of original, appellate and biofuels as a key to energy transition.
advisory jurisdictions.  It is also expected to contribute to job
 Supreme Court has been assigned the role creation and economic growth.
of a sentinel on the qui vive” as regards
 It was supported by G20 countries and
fundamental rights. The right to get redress
organisations like IAE, ICAO, WEF, and
from the Court is itself a fundamental right,
and the Court cannot abandon its own duty World LPG Foundation and on the
in this regard. sidelines of the latest G20 meeting at New
Delhi.
 The Global Biofuels Alliance was
85. Correct Option: (b) announced, bringing together biofuel
Explanation: producers and consumers with the intent
to strengthen global biofuels trade for a
 Statement 1 is incorrect: During the
emergency period, the Lok Sabha can greener sustainable future.
extend tenure by a period of 1 year at a time.  To ensure energy security, affordability,
But the same cannot be extended beyond and accessibility for the future, the
6 months after the proclamation ceases to GBA will facilitate global collaboration,
operate. The tenure of State Assemblies can supporting the development and deployment
also be extended in the same manner. of sustainable biofuels.

37
PTS2024/AIOMT/102023/07
87. Correct Option: (a) 89. Correct Option: (d)
Explanation: Explanation:
 Statement 2 is incorrect: India invites  Statement 1 is incorrect: The ‘Ayushman
all G-20 countries to join this initiative. Yet Bhav’ campaign aims to redefine healthcare
all the countries of Global south have not accessibility and inclusivity across the
joined the mission. country, and embodies a whole-of-nation
Supplementary notes: and whole-of-society approach.

G-20 Satellite Mission for Environment Supplementary notes:


and Climate Observation
Ayushman Bhava’ Programme
 During the G20 Leaders’ Summit in New
 As part of the Seva Pakhwada, Ayushman
Delhi, Prime Minister Narendra Modi
proposed the launch of a ‘G20 Satellite Bhav Abhiyan for saturation of essential
Mission for Environment and Climate health services is being held across the
Observation. ‘ country.
 The mission aims to provide climate and  The ‘Ayushman Bhav’ campaign inaugurated
weather data to countries, particularly by the Hon’ble President, Smt. Droupadi
those in the Global South. Murmu, which aims to redefine healthcare
accessibility and inclusivity across the

RE
 India during the summit mentioned that the
G20 satellite mission would be beneficial for country, and embodies a whole-of-nation
all of humanity just as the data obtained and whole-of-society approach.
from India’s successful Chandrayaan moon  The core objective of the Sewa Pakhwada is to
mission. extend comprehensive healthcare coverage
 The climate and weather data obtained from to every village and town, transcending
this will be shared with all the countries, geographical barriers and ensuring that no
especially the countries of the Global one is left behind.
O
South.
 As part of the Ayushman Bhav campaign,
 India invites all G-20 countries to join this in the last 3 days alone, over 2.5 lakh ABHA
initiative. IDs have been generated, over 10 lakh
people have received free drugs, and over
SC

88. Correct Option: (d) 8 lakh people have received free diagnostic
services.
Explanation:
 The Seva Pakhwada aims to play a pivotal
 All statements are correct. role in distributing Ayushman cards,
Supplementary notes: generating ABHA IDs, and raising awareness
about vital health schemes and disease
Delhi Police establishment Act conditions, such as non-communicable
GS

 The Delhi Special Police Establishment diseases, tuberculosis (Nikshay Mitra),


(DSPE) Act is an important piece of sickle cell disease, as well as blood donation
legislation in India, primarily known and organ donation drives.
for establishing the Central Bureau of
Investigation (CBI) and defining its
jurisdiction. 90. Correct Option: (b)
 The DSPE Act provides for the establishment Explanation:
of a special police force to investigate and
prevent corruption in the national capital  Statement 1 and 3 are correct: The
region. Peacock Soft-shelled Turtle as a critically
endangered species and mentions its native
 The DSPE Act empowers the Central
range, which is the Indian subcontinent.
Government to extend the jurisdiction of
this special police force to other Union  Statement 2 is incorrect: The Peacock
Territories as well. Soft-shelled Turtle is critically endangered
 The DSPE Act does provide the Central and is known for its soft, leathery shell,
Government with the authority to extend the primarily found in the rivers and wetlands
jurisdiction of the CBI to Union Territories of northern India.
other than Delhi.  Statement 4 is incorrect: They are
 This extension typically occurs through primarily carnivorous. They mainly
a notification issued by the Central feed on small fish, crustaceans, and aquatic
Government. insects.

PTS2024/AIOMT/102023/07 38
Supplementary notes: UAS) with primary and passive detection
means like radar, Radio Frequency (RF)
Peacock Soft-shelled Turtle (Nilssonia detectors, electro-optical, and infrared
hurum) cameras to be installed.
 They have a large head, and downturned  Mid-segment model: It will protect
snout with low and oval carapace of dark installations like metro airports, oil
olive green to nearly black, sometimes with refineries, ports, and power plants,
a yellow rim. etc A lower level of threat mitigation
 The head and limbs are olive-green; the techniques (compared to the full-scale
forehead has dark reticulations and large model) to be installed with Counter-
yellow or orange patches or spots, especially Unmanned Aircraft System(C-UAS).
behind the eyes and across the snout.  Basic model: Aims to protect state
 Males possess relatively longer and thicker secretariats, important official premises,
tails than females. monuments of national importance etc.
The basic threat mitigation technology
 Habitat: to be used.
 This species is confined to India,  In addition to the counter-rogue drone
Bangladesh, and Pakistan. deployment models, the civil aviation
 In India, it is widespread in the ministry has also suggested a set of legal
northern and central parts of the Indian procedures to handle rogue drones. The

RE
subcontinent. legislation needs to address the risk-
based use of Counter-UAS authority and
 These are found in rivers, streams, lakes, coordination among relevant departments
and ponds with mud or sand bottoms. and agencies.
 It shall also aim to mitigate adverse impacts
91. Correct Option: (d) of anti-drone guidelines on the safety,
efficiency, and accessibility to the Indian
Explanation: airspace to the maximum extent feasible.
 All statements are correct.
O  The law shall state a legal framework
for the authorized use of C-UAS systems
Supplementary notes: by security agencies for protecting vital
National Counter Rogue Drone assets, safeguarding manned aviation,
SC
supporting law enforcement activities,
Guidelines
protecting national borders, and
 The genesis of the counter-rogue drone conducting operations.
guidelines lies in the rising number of
 Radio Frequency (RF): It refers to the
drone-related safety incidents worldwide,
electromagnetic radio waves in the range
including in India. of 3 kHz to 300 GHz and the alternating
 Recent incidents of the utilization of drones currents carrying the radio signals.
to target VVIPs in Venezuela and Saudi
GS

Aramco drone attacks are a stark reminder


of the threat level from rogue drones. 92. Correct Option: (c)
India has an estimated 6 lakh rogue or Explanation:
unregulated Unmanned Aerial Vehicles
(UAVs)of various sizes and capacities.  Option (c) is correct.

 To handle the above scenario the Ministry Supplementary notes:


of Civil Aviation has suggested a counter- G-secs in a Bond market
rogue drone deployment plan, categorized
across three models, based on the sensitivity  Indian government bonds, also known as
of vital assets and installations. G-Secs or Government Securities, are debt
instruments issued by the Government
 The strategic installations differ from of India to raise funds for various public
state to state and place to place, based on expenditure and developmental projects.
their geographical condition, criticality,
 The issuer of Indian government bonds
and construction type but standard
is the Government of India through the
categorization would be:
Reserve Bank of India (RBI) acting as its
 Full-scale model: It will work for agent.
the protection of vital assets of critical
 These bonds are primarily issued to finance
national importance like Rashtrapati fiscal deficits, infrastructure development,
Bhawan, Parliament House, nuclear and various government programs. They
installations, major airports, etc; are a crucial source of capital for the
Counter-Unmanned Aircraft System(C- government.

39
PTS2024/AIOMT/102023/07
 Types: speech and voting in Parliament
 Indian government bonds come in various without interference from any court or
tenures, ranging from short-term (Treasury authority.
Bills) to long-term (Government Dated  MPs are immune from any legal proceedings
Securities or G-Secs). in respect of anything said or any vote given
 The most common G-Secs have maturities by them in Parliament or its committees.
of 5 years, 10 years, and 30 years. This immunity ensures that legislators can
express their views and cast their votes
 These bonds pay periodic interest to
bondholders, typically semi-annually. The without fear of legal consequences.
interest rates are determined through  Statement 2 is incorrect: When issues
auctions and can be fixed or floating. regarding the exercise of parliamentary
 Indian government bonds are actively privileges or legal challenges related to the
traded in the bond market. actions of MPs arise, the Attorney General
may provide legal opinions and advice
 They are listed on stock exchanges, and
to the Speaker of the Lok Sabha (House of
investors can buy and sell them in both the
primary and secondary markets. the People) or the Chairman of the Rajya
Sabha (Council of States) as needed.
Impacts on Economy
 Additionally, this immunity extends to
 Tendency for the currency to appreciate. certain non-members, like the Attorney

RE
 Increase in capital inflows into India. General of India or a Minister who
may not be a member but speaks in the
 Demand for investors to buy Indian
House.
government bonds denominated in Rupee
will increase.  In cases where a member oversteps or
 Increase in foreign portfolio investments exceeds the contours of admissible free
(FPI). speech, the Speaker of the House will deal
O
with it, as opposed to the court.
 Can lower inflation compared to other
economies. Supplementary notes:
Immunity from Bribery charges for MPs
93. Correct Option: (c)
SC
 Article 105(2) states, “No member of
Explanation: Parliament shall be liable to any proceedings
 Both statements are correct. in any court in respect of anything said or
any vote given by him in Parliament or any
Supplementary notes: committee thereof, and no person shall be
so liable in respect of the publication by
NASA’s OSIRIS-REx mission
or under the authority of either House of
 OSIRIS-REx (Origins, Spectral Parliament of any report, paper, votes or
Interpretation, Resource Identification,
GS

proceedings.”
Security, and Regolith Explorer) is a NASA
spacecraft launched in 2016 with the  Article 194(2) extends this immunity
primary mission of studying and collecting to MLAs and states, “No member of the
samples from an asteroid named Bennu. Legislature of a State shall be liable to
 The main goal of the OSIRIS-REx mission is any proceedings in any court in respect of
to collect samples from the asteroid Bennu anything said or any vote given by him in
and return them to Earth. the Legislature or any committee thereof,
and no person shall be so liable in respect of
 This will allow scientists to analyze the
the publication by or under the authority of
composition of the asteroid, providing
a House of such a Legislature of any report,
valuable insights into the early solar system
paper, votes, or proceedings.”
and the origins of life on Earth.

94. Correct Option: (b) 95. Correct Option: (a)


Explanation: Explanation:
 Statement 1 is correct: Article 105  Option (a) is correct: The Abraham
grants certain privileges and immunities Accords, signed in 2020, primarily focus on
to MPs, which are essential for the economic cooperation and normalization
effective functioning of the legislature. of diplomatic relations between Israel and
These privileges include freedom of several Arab nations in the Middle East

PTS2024/AIOMT/102023/07 40
Supplementary notes: Economically Backward Classes/De-notified
Tribes and Safai karamcharis including
Abraham Accords Waste pickers through online registration
 The Abraham Accords are a series of historic at PM-DAKSH portal on pan India basis.
agreements signed in 2020 that established
diplomatic normalization and economic
cooperation between Israel and several 97. Correct Option: (d)
Arab nations in the Middle East. Explanation:
 The key countries involved in these accords  Option (d) is correct: The term “Global
include Israel, the United Arab Emirates South” is used to refer to a group of
(UAE), Bahrain, Sudan, and later, countries primarily from Africa, Latin
Morocco. America, Asia, and Oceania, which are
 The Abraham Accords marked a significant generally characterized by their developing
shift in Middle Eastern diplomacy as they or emerging economies. These nations often
were aimed at improving relations between face common socio-economic challenges
Israel and its Arab neighbors. and seek a greater voice in global affairs,
advocating for their interests on various
 These agreements included the
international platforms.
establishment of diplomatic missions, the
exchange of ambassadors, and cooperation Supplementary notes:

RE
in various fields such as trade, tourism,
technology, and security. Global South

 They were seen as a way to promote regional  The term was first used in 1969 by the
stability and economic growth American academic Carl Oglesby.
 The term Global South was first used during
the Cold War to describe the broad economic
96. Correct Option: (a) divide between the wealthy, developed
countries of the North and the developing
Explanation:
O countries of the South.
 Statement 2 is incorrect: It is implemented
 The term also signifies a combination
by the three Corporations under the Ministry:
of political, geopolitical, and economic
 National Scheduled Castes Finance and similarities among nations.
SC
Development Corporation (NSFDC),
 The Global South includes countries that
 National Backward Classes Finance & are developing including countries in Africa,
Development Corporation (NBCFDC), Asia and Latin America.
 National Safai Karamcharis Finance and  Many of these were part of Western
Development Corporation (NSKFDC). European colonialism until the 1960s and
1970s. Nations in these regions typically
Supplementary notes:
experience higher levels of poverty, income
GS

PM-DAKSH (Pradhan Mantri Dakshta inequality, and challenging living conditions


Aur Kushalta Sampann Hitgrahi) compared to the wealthier nations of the
Global North.
 The Yojana is a revolutionary scheme and
national action plan to skill the marginalized
persons covering SCs, OBCs, EBCs, DNTs, 98. Correct Option: (c)
sanitation workers including waste pickers.
Explanation:
 It is a National Action Plan for skilling of
marginalized persons covering SCs, OBCs,  Both statements are correct.
EBCs, DNTs, Sanitation workers including Supplementary notes:
waste pickers.
 The main objective of the scheme is to Social Stock Exchange
increase the skill levels of the targeted  Stock exchanges are generally the
youth by providing for long term and short platforms where shares of companies after
term skills, followed by settlement in raising capital from public investors are
employment and self-employment. traded. Essentially a stock exchange listing
facilitates capital raising opportunities for
 The scheme has been launched by
the company.
the Department of Social Justice and
Empowerment during 2020-21 to impart  Investors get to buy and sell shares on such
skill development training to the youth exchanges. SSEs also mostly work the same
aged between 18-45 years belonging to way. Any not-for-profit organization (NPO)
Scheduled Castes, Other Backward Classes/ working for social goals essentially can raise

41
PTS2024/AIOMT/102023/07
capital from public investors by listing on phenomena are in the same phase, their
SSE akin to what normal companies do for associated impacts can be amplified. In
listing on stock exchanges. the opposite phase, the associated impacts
 However, a key difference between SSE on global climate may be reduced.
and a normal stock exchange is trading by
investors. Since the money being invested in
an SSE listed entity is a donation, investors
100. Correct Option: (b)
can neither make returns on such securities Explanation:
nor trade them like normal shares.
 Statement 3 is incorrect: India ratified
the UNTOC in 2011 becoming the fourth
99. Correct Option: (b) South Asian country to do so. The nodal
agency for all dealings with UNTOC is the
Explanation: Central Bureau of Investigation (CBI).
 Statement 3 is incorrect: Positive phases
Supplementary notes:
of the PDO tend to be associated with
periods of more rapid global warming whilst United Nations
cold PDO events have been linked to severe
 The United Nations Convention against
droughts spanning many years in south
Transnational Organized Crime (UNTOC,
western USA, as well as increased rainfall
often known as the Palermo Convention) is

RE
over eastern Australia.
a multinational treaty against transnational
 Statement 4 is incorrect: The Pacific organized crime that was established by the
Decadal Oscillation (PDO) is a long-term United Nations in 2000.
climate pattern that affects the temperature
 The United Nations Convention against
of the Pacific Ocean and the weather
Transnational Organized Crime, which
patterns around it.
was enacted by the General Assembly in
Supplementary notes: Resolution 55/25 on November 15, 2000,
O
is the most important international tool in
Pacific Decadal Oscillation (PDO) the battle against transnational organized
 The Pacific Decadal Oscillation (PDO) crime.
is a climatic event which covers vast areas  It was signed by the Member States at
SC
of the Pacific Ocean over periods of 20 to a High-level Political Conference held in
30 years. Palermo, Italy, on December 12-15, 2000,
 The PDO has positive and negative phases. and came into force on September 29, 2003.
The climate impacts experienced during Why in news?
a PDO event can go hand-in-hand with  20 years of United Nations Convention
impacts from El Nino or La Nina. If both against Transnational Organized Crime.
GS

™™™™™

PTS2024/AIOMT/102023/07 42

You might also like